Você está na página 1de 370
Loar ge Gleason A Cer TT Instructor’s Solutions Manual SINGLE VARIABLE Third CTC UL'UIS INSTRUCTOR’S SOLUTIONS MANUAL to accompany CALCULUS ee SINGLE VARIABLE THIRD EDITION Deborah Hughes-Hallett University of Arizona Andrew M. Gleason Harvard University William G. McCallum University of Arizona etal Prepared by Ray Cannon Srdjan Divac Elliot J. Marks Emie S. Solheid Xianbao Xu JOHN WILEY & SONS, INC. New York * Chichester + Weinheim + Brisbane + Singapore + Toronto COVER PHOTO © Eddie Hironaka/The Image Bank. To order books or for customer service call 1-800-CALL-WILEY (225-5945). Copyright © 2002 by John Wiley & Sons. Ine. Excerpis from this work may be reproduced by instructors for distribution on a not-for-profit basis for testing or instructional purposes only to students enrolled in courses for which the textbook has been adopted. Any other reproduction or translation of this work beyond that permitted by Sections 107 or 108 of the 1976 United States Copyright Act without the permission of the copyright owner is unlawful Requests for permission or further information should be addressed to the Permissions Department, John Wiley & Sons, Inc., 605 Third Avenue, New York, NY 10158-0012. ISBN 0-471-44185-6 Printed in the United States of America 10987654321 Printed and bound by Bradford & Bigelow Ine CONTENTS CHAPTER 1 oo... eee cece eee 1 CHAPTER 2 2.2... 0.00 e eee eee 53 CHAPTER 3 «2.0.2... 0 eee eee ee cee cece ee 107 CHAPTER 4 .. 185 CHAPTER 5 275 CHAPTER 6 313 CHAPTER 7 0... cece cece e cece eee » 355 CHAPTER 8 2.0... 0.00... cece ee eee cece eee 457 CHAPTER 9 «0.0... eee eee eee cece eee eee 543 CHATTER O cheesy cee eee eee 575 CHAPTER 11.......... 2000 c cece ee eeee eee 641 APPENDIX... 0... ee cece eee 735 ‘1souumons 1 CHAPTER ONE Solutions for Section 1.1 Exercises 1. 4(38) means the value of P coresponding tof = 35. Since f represents the numberof years since 1950, we see tht (88) means the population ofthe city in 1985. So, n 1988 the city’s populition wax 12 milion 2 Since T = (P), we see that f(200) isthe value of T when P = 200: that is, he thickness of pecan eggs when the concentration of PCBs is 200 ppm. 3. (a) When the crs 5 years oi, itis worth $6000. {b) Since the value ofthe car decreases asthe ear ges older. this isa decreasing Function. A possible graph sin Figure 1.1 rs as = as Figure 14 {€) The vertical intercept isthe value of 1” when a = 0, or the value ofthe car when i is new. The horizontal intercept is the value ofa when = 0, or the age ofthe car when iis worth nothing. 4. (a) The flat ire corresponds to part of the graph where the velocity is zero. This could be (II) or (IU), Since the velocity in (II) is higher forthe later part ofthe interval, corresponding to speeding up, the answer is (ID). (b) This i (1), as the graph shows a postive velocity followed by a zero velocity (@) In (IV), the velocity is positive, zero (while the package is being dropped off) and then nepatve (the drive hom (a) Groph (Ill) could represent a drive to the country, stopping to have lunch (velocity zero), and continuing on at a slower speed (0 look at the scenery. Other stories are possible Ps Rewriting the equation as, shows thatthe line has slope ~12/7 and vertical intercept 2/7 6, Rewriting the equation ofthe line as We see the line has slope 1/2 and vertical intercept 7. Rewriting the equation ofthe line as y 2 watz 3 ‘we see thatthe line has slope 2 and vertical intercept ~2/3. 8 The slope is (1 — 0)/(1 ~ 0) = 1. So the equation ofthe line is y = + 9, The slope is (3 — 2)/(2 ~ 0) = 1/2. So the equation of the line is y = (1/2)2 + 2 2 Chapter One SOLUTIONS 10, Using the points (~2. 1) and (2,3). we have 1 TCH AT Now we know that y= (1/2) +b Using the point (~2,1), we have 1 = ~2/2 + by which yields b = 2. Ths, the esuation ofthe tine sy = (1/2) +2. Sr ~3. Since the slope ofthis lines 5. we want ane == —He-2eory= he F 12, The line y +r = 7 has slope A, Therefore the parle! line has slope — and equation y ~ 5 = A(z ~ 1) or “iz + 9, The peponicolr line has slope ty = 4 and equation y 5 = 1(2— I) ory = 0252 +4. 13, ‘The line parle to y= mz + cals has slope, 3 is equation is Slope = th slope ~ } passing through the point (2.1). The equation ysl a) +6. “The line perpendicular to y = ynz + ebas slope ~1/ni, so its equation will be 1 y=-H(e~a) +6. 14, (a) is (V), because slope is positive. vertical intercept i negative (hy is (LV), because slope is negative, vertical intercept is positive {e) is (), because slope is 0, vertical intercept is postive (2) is (V1), because slope and venieal intercept ate boah negative (c) is), because slope and vertical intercept are both positive (fis AID, because slope is positive, vertical itercepr is 0 15, (a) is (V), because slope is negative, vertical intercept is 0 (b) is (VD, because slope and venical intercept are beth positive (c) is () because slope is negative, venicalimercept is postive (@) is UV), hecause slope is positive, vertical itercep is negative (e) is ID), because slope and vertical intercept are heth negative (8 is (I), because slope is positive, vertical intercept is 16, Since the function goes from 2 OSs. Since the function goes from = ~2Sys2 18, Since + goes from I 10 Sand y goes from | 106, the domain is 1 < x < 6 and the rangeis 1 2, since x* > @ forall 2, tox = 5 and beween y = Oand y the domain is 0 < x < 5 and the range is 2wor = 2and fom y 20 2,the domain is -2 < x < 2 and the range is 20, Te doesnt al > th ania ase em Theregisd <9 The value of f(t) is real provided f? — 16 > 0 oF /# > 16. This occurs when ether t > 4, ot < —4. Solving f(t) = 3, we have VEG 22, Factoring gives e-ne+y “Ten and x = —1. when the denominator is 0. So the domain is all wor's factors is 0, s0 2 = 22. ote) = ‘The values of z which make g(2) undefined are £0,—1, Solving y() = 0 means one of the nun For some constant k, we have $= KR? sasowmnons 3 24, We know that Eis proportional to v2, so for some constant k 28, We know that is proportional to 1/F", so A. foesome constant Problems x owes | | | 27, (a) f(B0) = 10 means thatthe value of fat = 30 was 10. In oer Words, the temperature atime t= 30 nutes was 10°C. So, 30 minutes afer the object was placed outside. it had cooled 10 10°C, {b) The intercept a measures the value of f(4) wien f= O. In other words, when the object was intially put ouside, it had a temperature of a°C. The intercept b measures the value of € when f(t) = 0. In other words, at time b the ‘object's temperature is 0°C. 28, sven) | | ime) ». Detar tom Kaname 158 120 5 ——~ twe sain amen re a Grea KSbmano ‘eve [M4 (a) Wend the slope m and inercept bin he ner equation © BC, Bom Se = 0100 b+ me."To find the slope m, we use 02. We substitute to find b “The linear formula is C ‘Chapter One SOLUTIONS () The slope i 0.2 dollars per kilogram. Each adcitional kilogram of waste costs 20 cents. (6) The intercept is 12 dollars, The flat monthly fee to subscribe to the waste collection servic is $12. This isthe amount charged even if there is no waste, as 103 ~ 90 7 3te (a) Charge per ebiefowt= 5S = B= BI = sunrise. ‘Alternatively, if we let & = e0st, wv = eubie feet of water, b = fixed chary ++ mar. Substituting the information given inthe problem, we have 96 = b+ 1000 105 = b+ 1600. ind m = cosveubie fet, we obtain Subtracting the frst equation from the second yields 15 = 600m. som = 0.025. (b) =b + 0.0251, so 90 = b + 0.025(1000), which yields b = 65. Thus the equation isc (©) We need to solve the equation 130 = 65 + 0.0251, which yields w = 2600. 32. (a) Given the two points (01,32) and (100, 212) and assunning the graph in Figure 1.2 isa line, 65 + 0.025u: spe = 22=82 18g | *f we Wo 130 200 Figure 1.2 (b) The °F intercept is (0,32). 0 °F = 18°C) +32, (6) Ifthe temperature is 20°Celsius, then F = 1.8(20) +32 = 68°F (a) 16°F = °C, then *cH18°C+92 32 = 08°C "c= 40" 33, (a) This could be a linear funetion because w increases by 5 as increases by. (b) We find the slope m and the intercept b inthe linear equation w = b + mb, We frst find the slope vn using the frst two points in the table. Since we want 1 tobe function off, we take Aw _ 171-166 Sn > wos 5 into the Hinear equation Substituting the fest point and the slope sob = ~174. The linear function is + mah, we have 166 = b + (5)(68). sh 174, The slope, m1 = 5 isin units of pounds per ined, b+ mw using m = \h/Aa to obtain the linear function +348, Alternatively, we vould solve the linear equation found in par (b) For. The slope. m pound. 2, has units inches per sasoLuTIONS «5 3M. We are looking fora linear function y = f(x) that, giver atime x in years. gives a value y in dollars for the value ofthe refrigerator. We know that when 2 = 0, tha is, when the refrigerator is news, y = 950, and when x = 7, the refrigerator is worthless, so y = 0. Thus (0,950) and (7.0) are on th fine that we are looking for. The slope is then given by 950 7 1s negative, indicating thatthe value decreases as time passes. Having found the slope, we can take the point (7.0) and use the point ‘slope formula: yam = m(e~ 21), So, 35. (a) We have the following functions. (i) Since a change inp of $5 results in a decrease in of 2, the slope of g = D(p) is ~2/5 items per datar. So Now we know that when p = 530 we have g 100 =~ 2 too = 6 ~ 220 320 Thus a formula is = 320- Gi) We ean solve q = 320 ~ Fp for pin terms og iy = 1600 2p 2p = 1600 ~ 59 p= 800 ~ 34. The slope of this fretion i 5/2 dlls per items we would expec. (by A gph ofp = 800 ~ ie given in Fique 1. iets) 00 1 es) io a0 Figure 13 5 Chapter One ‘SOLUTIONS. 36, Given I = lo = alot ~ fo) with fo to and a all constant. (a) Weave I = alo(t — to) + lo = alot — aloto + fo, which isa linear function oft with slope alp and y-intercept at (0. -aloto +10) (b) Ilo = 100. 60°F and a = 10~*, then 107% + 99.91 0.001" + 99.94 1= 107*(100)" ~ 107*(100)(60) + 100 {c) Ifthe slope is positive, (as in (D), then as the temperature rises. the length of the metal increases: it expands. If the slope were negative, then the metal would contract asthe temper a1. (a) R k(950 — H), where kis a postive com IF His greater than 350° the rate is negative, indicating that s very hot yam will coo! down toward the temper ature ofthe oven. (b) Letting Ho equal the inital temperature ofthe yam the uraph of Fugainst F looks lke: R (350 ~ Ho) le 0 temperature ofthe yam, since the yams surface will [Note that by the emperarure of the yam, we mean the aves bbe hotter than its center. (a) Assembling the given information, we have img — ke) where kis the constant that relates velocity to sir resistance (which depends on the shape of the object), (b) Solving the above equation for a we have © a 39, Looking atthe given data, it seems that Galile’s hypothesis was incorrect, The ist tle suggests that velocity is not 8 linear funetion of distance, since the increases in velocity for each foot of distance are themselves geting smaller. Moreover, the second table suggests thit velocity is instead proportional to rime, since for each second of time, the velocity increases by 32 sec. Solutions for Section 4.2 —_——_——————— Exercises 1. The graph shows a concave up function 2 The graph shows a concave down funetion. 12sounoNs 7 3. This graph is nether concave up or down, 4. The graph is concave up. 7. Initial quantity 3% (eominwus) 8, ital quantity = 15; growth rate ~6% (continuous deeay. 9. (a) The Function is linear with initial population of 1000 and slope of 5D, so P = 1000 + 30t () This funtion is exponential wit initial population of 1000 and growth rate of SS, so P = 1000( 1.05)" 10. (a) This isa linear function with slope ~2 grams per day and intercept 30 grams. The function is Q = 30-~ 2t, and the graph is shown in Figure 1.4 tons) Qigams 0 0 30(0.88)" S— ae ta Figure 14 Figure 1.5 (b) Since the quantity i decreasing by a constant percent change, ths isan exponential function with base 1 ~ 0.12 (0.88, The function is Q = 30(0.88)', and the graph is shown in Figure 1.5. 11, The function is increasing and concave up on the 4-interal between D and F and the z-interval between H and I. Its increasing and concave down on the 2-interval between .A and B. and the x-interval between E and F. Ibis decreasing land concave up on the z-interval between C' nd D, and the 2-interval bewween G and H. Finally itis decreasing and concave down onthe 2-interval between B and C, and the x interval between F and G Problems 2 @) astm (b) “The rate at which new people ty it” isthe rate of change of the total number of people who have tried the product ‘Thus the statement of the problem is tlling you thatthe graph is concave down—the slope ix postive but deereasing, asthe graph shows, 13, (a) Advertising is generally cheaper in bulk: spending more money will give bewer and better marginal results initially, (Gpending $5.000 could give you a big newspaper ad eaching 200,000 people: spending $100,000 could give you 3 series of TV spots reaching 50.000,000 people.) A graph is shown below lef (b) The temperature of a hot object decreases at arate proportional to the difference between its temperature and the temperature of the ait around it, Thus, the temperature of «| very hot abject decreases more quickly than a cooler ‘object. The graph is decreasing and concave up. (We are assuming that the coffe i al a the same temperatute.) M (Graper One SOLUTIONS revenue temperature NK advenising time (a) A linear function must change by exacily the same amount whenever x changes by some fixed quantity. While h(2) decreases by 3 whenever x increases by 1, f(x) and g(2) fail this tet, since both change by different amounts between x = 2 and x = —1 and between 2 = —1 and z = 0, So the only linear function is A), so it will be given by a formula of the type: A(2) = mz +b. Asnoied, m = —3. Since the y-intercep of his 31, the formula for Nx) is h(a) = 31 ~ 32. (b) An exponential function must grow by exactly the same factor whenever x changes by some fixed quantity. Here, 9(z) increases by a factor of 1.3 whenever x increases by 1, Since the y-intercept of g(2) is 36 g(2) has the formula 9() = 36(1.5"). The other two functions are not exponential: h(x) is not because itis a linear function, and f(z) ‘snot because it both increases and decreases. 1S, (a) Let P represent the population of the world, and let & represent the number of years since 1999. Then we have P= 6(L.013)'. (b) According 0th billion people {(€) The graph is shown in Figuee | {his oceurs a approximately 53.7 years, formula. the population of the world inthe year 2020 (at € = 21) will be P = 6(1.018)" = 7.87 ‘The population of the world has doubled when P 3.7. Under these sumptions, the doublis 2; we sce onthe graph that {ime ofthe world's population is about ‘ 1 tens 16, (a) We have Reduced size = (0.80) - Original size Original size = Reduced size 1 : aaa (1.25) Reduced size, so the copy must be enlarged by a factor of 1.25, which means itis enlarged to 1256 of the reduced size (b) 1a page is copied n times. then New size = (0.80)" - Original ‘We want t0 solve for 50 that oso)" 0.168 and (0.80)? Sy tal and ero, we find (0.80)* 12sowumoNs «9 17. (a) Using Q = Qo(1 ~ 1)! for toss, we have Q ‘The investment was worth $3486.78 alter 10 years (b) Measuring time from the moment at which the stock begins to gain value and letting Qo ft yearsis 0,000(1 - 0.1)"° = 10,000(0.9)"° = 3486.78. 3486.78, the value after Q = 3480.78(1 +0.1)' = 3486.78(1.1)" ‘We can estimate the value of ¢ when @ = 10.000 by tracing along a graph of Q, gi years fo get the investment back to $10.000. 18, We look for an equation ofthe form y = yod" since the graph looks exponential. The points (0,3) and (2.12) are on the raph, so 11. Teil take about 11 mE. we and a? giving a = 42, Since a > 0, our equation is y (24), 19, We Took for an equation ofthe form y = yoa* since the graph looks exponential. The points (~1.8) and (1.2) are on the graph, so Hence y = 4(3) We look for an equation of the form ‘raph. so 1yoa™ since the graph looks exponential. The points (1.6) and (2.18) ae on the yoo! and 18 = yon? ast = 18 = 5, ands06 = yoa = yo S:thus, yo = 2. Hence 21, The difference, D, bewe equation of the form ‘Therefore a 23"). the horizontal asymplote and the graph appears vo decrease exponentially, so we Took for an D=Da* where Do = 4 = difference when x = 0. Since D = 4 ~ y.we have 4aysda% or y=4—4a% = 401-0") ‘The point (1,2) is ont Therefore y = 4(1 = (4)* 22 (a) Let Q = Qua! Then Qoa® = e graph, 302 = 4(1 — a"), giving a = 3 4-2), 73.94 and Qua? = 170.86, So Qoa? _ 17088 _ 9 95 Goat = Fa.gr = 228 Soa=15 (b) Since a = 1.5. the growth ate sr = 0.5 = 50% 2B, (a) Let Q = Qon', Then Qoa%"? = 25.02 and Qou®®* = 25.06, So Qua? _ 25.06 Gout? = 35.03 = NOE = al So 3.001) = 1.05. (b) Since a = 1.05, the growth rate is r = 0.05 = 5%. 2b Since e928 = (e#8) = 1.28", we have see that this is growth because 1.28 > 1 Since e®* = (0°8*)! = (0.61)', we have P also see tht this is decay because 0.61 < 1 15(1.28) This is exponential growth since 0.25 is postive. We ean also 2(0.61). This is exponential decay since ~0.5 is negative, We can 10 Chapter One ‘SOLUTIONS 26, P = Po(e®2)' = Po(1.2214)'. Exponential growth because 0.2 > 0 or 1.2214 > 1, 21, P= Tem")! = 7(0.0482)'. Exponential decay because —7 < 0 oF 0.0492 < 1. 28, (a) We have Py = 1 million, and k = 002, so P = (1.000,000)(¢°°") P 1,900,000 ‘ Direct calculation reveals that each 1000 foot increase in aude results ina longer takeoff roll bya factor of about 1.096, Since the value of d when ht = 0 (sea level) isd = 670, we are led tothe formula = 67041.096)"", ‘where di the takeoff rollin feet, and his the airport's elevation, in fet. Alternatively, we can write d= doa, where dy isthe sea level value of d, do = 670. In addition, when 734 = 670'™, Solving fora gives 30, Since fis lin its slope is constant “Thus f increases 5 units or unit inerease in 2, so JQ) =15, f(@) =25. 30, Since g is exponential its growth factor is constant, Writing 9(2) = ab*, we have g(0) = a = 10,50 gz) = 10-8", Since 9(2) and since 8 > 0, we have vi. ‘Thus g increases by a factor of V3 for unit increase in 7,50. g(t) = 10V3, (3) = 10(V3)° = 20V3._g(4) = 10(V3)* = 40. [Notice that the value of g(z) doubles between z = and x = 2 (from g(0) = 10 to g(2) = 20), so the doubling time of ‘9(z) is 2. Thus, o(z) doubles again between x = 2 and x = 4, confirming that g(4) = 40, 31 If the pressure at sea level is Py, the pressure P at altitude his given by p=n(i-2t)™ since we want the pressure to be multiplied by a factor of (1 ~ $34) = 0.996 for each 100 feet we go up to make it decrease by 0.4% over that interval. At Mexico City h = 7340, so the pressure is P = Po(0.096) 7H ~ 0.745 Pp, So the pressure is reduced from Py to approximately 0,745 Po, a decrease of 25.5%. s2souumoNs 11 32, (a) We compound the daily ination rate 30 times to get the deste monthly rater ry! 13)" (+35) = 0+ an) Solving forr. we get r = 47.3, s0 the inflation rate for June was 47.39% (b) We compound the daly inflation cate 365 times to get a yearly rule RE Ry 1a" (1+ 56) = (1+ jaa) Solving for Rw gt Ft = 110.544, so the yearly rate was 1154.49 dring 1988. We could have obtined approx imately the same result by compounding the monthly rate 12 times. Computing the annual rae from the monthly gives a lower result, because 12 months of 30 days each is only 360 days, ‘3%. (a) Thisisthe graph of a incar function, which increases a a constant rate, and thus corresponds o h(t) which increases, by 0.3 over each interval of 1 (b) This graph is concave down, soit corresponds toa function whose increases are getting smaller, as isthe ease with ‘A(t. whose increases are 10.9, 8,7, and 6. (€) This graph is concave up, soit corresponds 10 a function whose increases ae getting bigger. a isthe case with g(t). whose inereases are 1, 2,3, 4, and 5. 3M. The doubli time ¢ depends only on the growth rate; it isthe solution to 2.02)" Since 1,02* represenis the facior by which the population has grown after time f. Trial and error shows that (1.02)°° = 1.9999 and (1.02)°* = 2.0390, so tha the doubling time is about 35 years. 35. (a) The forma is Q = Qo(3)°. (b) The percentage left after 500 years is 20 The Qos cancel giving ()" xaser, 0 80.7% is let 36. Let Qo be the intial quantity absorbed in 1960, Then the quantity, Q of stronium90 left after # years is ona ey” ‘Since 1990 ~ 1960 = 30 years elapsed, the fraction of strontum-$0 ett in 1990 is yor Qo(F @ Q (y= a8 = 88%, 37, Because the population is growing exponentially, the time it takes to double is the same, regardless of the population levels we are considering. For example, the population is 20.000 at time 3.7, and 40,000 at time 6.0. This represents a doubling ofthe population in a span of 6.0 ~ 3.7 = 2.3 years. How long does it take the population to double a second time, from 40,000 to 80,000? Looking atthe graph once again, we sec that the population reaches 80,000 at time t = 8.3. This second doubling has taken 8.3 ~ 6.0 = 2.3 years, the same amount of time asthe frst doubling Further comparison of any wo populations on this graph that differ by a factor of two will show thatthe time that separates them is 2.3 years. Similarly, during any 2.3 year period, the population will double. Thus, the doubling time is 23 year, ‘Suppose P = Poa doubles from time #10 time t+ d. We now have Poa! 30 Poa! = 2Pya Thus, canceling Py and ad mst be the umber such that a4 = 2, no matter wht ts 12 Chapter One ‘SOLUTIONS 38, We see that 198 = 1.03, aml therefore M(s) = o(1.09):€ must be 1. Similarly $85 = 1.1 and so f(s) = a(t)": a= 2. Lastly, BB 1 (1.05): b= 3 39, (a) The slope is given by Poh 100 T=h ~ 20-0 ~ 20 We know 30 when t= 0, so P= 251-430, () Given P = Poa! and 50 when t 30 = Pa”, s0 Po Then, using P = 100 when ¢ = 20 100 = 50a°® 2e0* a= 2 = 1.035265, And so we have P= 50(1.085268) ‘The competed abies found in Table (© ic st 200 10 > 160 Table 1A The cost ofa home fa ‘e)Linew Growth_] 5) Exponential Growth 10 # | rien sion0 units) | rien 1000 ui 400 ° 30 30 2 10 78 ot ® 2 100 100 © x0 rs vai laa * 150 200 o5 wis we eM () Since economic growth (inflation, investments) are usually measured in percentage change per year, the exponential ‘model is probably more realistic. Solutions for Section 1.3 Exercises (a) gQ4h)= +h 4+2Q4h) +344 4h4 444 WEI = He OH T (by 9(2) = 2422) += 4444 (©) 92+) ~ 9(2) 11, which agrees with what we get by substituting h = 0 imo (a). (#4 6h +11) ~ (11) = 4? + 6h, Parse ae 4m42 eR La Lag DF 2, (8) 2f(t) = 20 +1) = 2 +2. fe) (f@) +1 = (P41) F128 HeLa t+ oe 42 3 (a) f(a) + 9(n) = (Bn? — 2) + (m1) = Sn? +9 — (b) fladgln) = (Gn? — 2)(n +1) = Bn? + Sn? — 2-2. (©) the domain of HE? is defined everywhere where gn) #0. for al Holm) = 30 1) = 2 = 308 + 6 +L fe) of f(n)) = Gn? — 2) + 1 = 3n? = 1. tssownions 13, 4. Figure 1.7 shows the appropriate graphs. Note that asymmprotes are shown as dashed lines and a-or y-intercepts are shown as filled circles « y o 8, m(z+h) = m(z—h) + Ohz +h? - (2? - the +h?) 9. (a) £(25) is qcomesponding to p = 25, of i other words, he number of tems sold when he pic is 25. (b) f~1(30) is p corresponding to g = 30, or the price at which 30 units will be sold. 10, (a) $(10,000) represents te value of C eoresponding to A = 10,000, orn other word the cos of building & 10.000, Squatetoot stor. (b) f—#(20.000) represents the value of A corresponding to C = 20.000, or the area in square feet of a store which ‘would cost $20,000 to build LL, £71(75) is the length of the columa of mercury in the thermometer when the temperature is 75°F. 12, (a) Since m = f(A), we se that f(100) represents the value of m when A = 100. Thus f(100) isthe minimum annus {70s income needed (in thousands) take out a 80-year mortgage loan of $100,000 at an intrest rate of 9% (b) Since m = f(A), we have A = f~'(m). We sce that f~*(75) represents the value of A when m = 75, or the size of a morgage lin that could be obzined on an income of $75,000, 13, The faction is not invertible since thee are many horizontal ines which hit the Function twice 14, The Function isnot invertible since there are horizontal lines which bit dhe function more than once. 15, (a) We find f~"(2) by finding the z value corresponding to f(r) = 2. Looking atthe graph, we see that f-1(2) = =1. (b) We construct the graph of f~(x) by reflecting the graph of f(x) over the line y = =. The graphs of f—!(x) and f(z) ace shown together in Figure 1.8. Figure 1.8 16, This looks ikea shift of the graph y = —2". The graph is shifted to the left | unit and up 3 units, so a possible formu isy=-(@ 41) +8 14 chapter One ‘SOLUTIONS 17, This looks lke a shift ofthe graph y = 2%, The graph is shifted to the right 2 units and down 1 unit sa possible formula isy=(r- 2-1. 18, Soa) = (-2)8 + (-2)t 1s 2-28 $1. Since J(—2) # fle) and f(—2) # ~ f(z), this function is nether even nor odd » f(-2) = (-2)? + (-2)* + (-2) Since J(—2) # Fle) and f(—2) # ~ (2), this function is nether even nor od Problems 20, (a) The equation is y = 2z* + 1. Note that its graph is narrower than the graph of y = 2? which appears in grey. See Figure 1.9, 8 yee 8 = 27+) t 7 6 6 4 yaa \\a a \2 Kit Figure 19 Figure 1.10 (0) y= 2(24 +1) moves the graph up one unit and then stretches it by a factor of two. See Figute 1.10. (6) No, the graphs are not the same, Note that stretching vertically eaves any point whose yale i zero in the same place but moves any ether point. This isthe source ofthe difference because if you stretch it irs its lowest point stays atthe origin Then you shift tp by one and is lowest points (0,1) ARermatively, if you shift fis, is lowest ‘point is (0.1) which, when stretched by 2, becomes (0,2). 21, Not invenble. Given a certain number of customers, say f(f) = 1900, there could be many times , during the day at which that many people were in the store, So we dont know which time instant the right one. 22, Probably not invertible. Since your calculus elas probably has less than 363 students, there willbe at leat two days in the year, say @and , with f(a) = f(0) = 0. Hence we don’t know what to choose for f"(0). 23, Invenibe. Since at 4°C, the mass of 1 liter of waters 1 kilogram, the mass of 2 Titers is x kilograms, So f(z therefore, f-"(2) = 2. 24. Not invertible, since it costs the same to mail a SO-gram ltr a it does to mail 51-gram letter. 28. flo(0) = f2) = 0. 26, g(F2)) = (04) LA. 27. {(F() = f(-0.4) = -039. 28, Computing f(g(2)) asin Problem 25, we get the following table. From it we graph f(g(2)). and Ce =05 aa a2 a2 1s -05 =02 ot 07 oul Sata) sasownions 15 29, Using the same way to compute 9(f(2)) as in Problem 26, we get the following table. Then we ean plot the graph of a(f(2)). = [ie) To 7) =26 or | os at ai} maa TT wen -15|-13] —18 I mi fava) az ' wos} —1 | -14 : o |-o8) - < 3 as |-08] -08 4 1 |-04} “03 15 |-01] 03 2|oa| i as}oo} 2 3 [is | 22 30, Using the sime way to compute #(/(z)) as in Problem 27, we get the following table. Then we can plot the graph of HG). = [eG yay le -25} 01 | -07 ts | ee 15] -13 1@) ar |a2 es nos! -1 0 |-o8 | as |-o8 = 1 | -os Tt 1s | -01 Ty t 03 a 09 16 BL flz)= 2°. g(z)=2 +1. 32 fle 2 rth gz)=2. 33. Since B= y — 1 and n = 25? ~ B, substitution gives B® — B= Ay - 1)? ~ (y—1) = 29? Sy +3. ‘M4. (a) The function f tells us C in terms of q, To get its inverse, we want qin terms of C, which we find by solving for 4 00 + 2e, re). (b) The inverse function tells us the number of articles that can be produced for a given cos 135. (a) For each 2.2 pounds of weight the object has, it has 1 kilogram of mass, so the conversion formula is (b) The inverse Function is and it gives the weight of an object in pounds as a function of its mass in kilograms. 16 Chapter One ‘SOLUTIONS 36. Solutions for Section 1.4 —————— Exercises 1, The function e* has a vertical intercept of 1, so must be A. The function nx has an x-intercept of 1, 0 must be D. The ‘graphs of 2 and z*/* go through the origin. The graph of 2? is concave down so it corresponds to graph C and the raph of 2? is concave up so it corresponds to graph B. . Taking logs of both sides log 3° = rlog.3 = log 11 ‘3. Taking logs ofboth sides log 17 = log? 2g 17 = log 2 108% 2 9.24, jog, ‘Taking logs ofboth si log 10 = log 4? = rlog 4 ogo | 1 Tost fog * 1°6 20 = 50(1.09)" 1.04)" 20 50 Taking logs of both sides log log(1.04)* log? = 2og( 1.04) log(2/5) Tou(7.03) ~ 28.4 46, Taking logs of oth sides sasowuTioNs 17 ‘Taking logs ofboth sides tos (7) = 108 (3) (4/7) Ho (5/3) ‘Taking logs ofboth sides 9. In(2*) = Infe"*) rin? =(2+1)Ine zln2=2+1 0.6932 = 2 +1 1 oes: 10, To solve for x, we fist divide both sides by $ and then take the natural logarithm of both sides 12, In(2e%*) = In(4e**) in2+In(et*) = In + In(e™*) 0.693 + 32 = 1.386 +52 2 = -0347 18 Chapter One ‘SOLUTIONS 13, Using the rules for In, we get 7 (24 2)n7 = 172 (iu 7-17) = -21n7 1 n(10°*) = Ine (c+3)n0 =ln5+(7—2)Ine 2:308(2 +3) = 1.609 + (7-2) 330082 = 1.609 +7 ~ 2.203(3) 2 = 0515 1S. Using the rules for in, we have te-1 Hats1=0 fe-F 16, Using the rules for In, we get Ing = In2et rind =In242* zing +In2 = 0. [tao Tad Wn9s VUNOE AH? or 0382.2 = 1.815. We can use the quadratic formula to get 2 loga v= eS ae, = BC) _ log P - log Po ' log P — log Po Toga Toga 19, Taking logs of both sides yields Tog Hence Yon ($) _ log @ = logQo loge ‘loga 20, Collecting similar terms yields Hence = nk eee met SK, ankaneor="d 23, Using the identity” z,we have 0) sasownons 19 24 Using the identity 10!°** = x, we have 10848) AB 25, Using the identity e™* = x, we have 5A, 26, Using the identity In (e*) = 2, we have 2AB. 27, Using the rules for in, we have in(2) +inap=iat-mnetind tin O-14+lnayinB 1+ leat in B. 28, Using the rules for In, we have 2.4 + Sein B. (ele) = lint = "4, we have P = 152", Since 0.41 is positive, this is exponential growth, 28, Since (1.5)! (0.5306, Thus P = 106% 0.1054, Thus P 42-980. Since —0.6 is negative, tis represents exponential decay: 30, We want 1.7" 31, We want 0.9 32, Since (0.55) ie :a088e (ete) lesan, 33. If p(t) = (.04!, then, for p™? the inverse ofp, oe should have our =e, P(t) log( 1.04) = log, 1) = lost Ph O* Bao 58.708 logt. BM. Since fis increasing, fas an inverse, To fn the inverse of J(0) = 50%, we replace ¢ with J-"(0), and, since FO) = we have ta auc", We then solve for f= "(t) 0070) orn EC 35. Using f(F-"(0) tewe see AO) = 14 nfO So wero 1 rose Problems 136, We know thatthe yintercept ofthe Tine ist (0,1), so We need one other point wo determine the equation ofthe line. We ‘observe that it intersects the graph of f(z) = 10* atthe point x = log 2. The y-coordinate of tis point is then ple of 20 Chapter One ‘SOLUTIONS s0 (log 2.2) isthe point of intersection, We can now fing the slope ofthe line: a1 ™ = i920 ~ log? Plugging tis into the pointstape formula for a Tine, we have 1 ~ 0.18, the decay rate is 0.18, so 186 leaves the body each hour, (0(0.82)* = 3.04. The amount in the body after 6 hours is 3.04 me. (4) We want to find the value of ¢ when A = 1. Using logarithms: 10(0.82) 01 = (0382)' tq.) = €1n(0.82) = 11.60 hours After 11.60 hours, the amout is 1 mg. 38. (a) Since the initial amount of caffeine is 100 mg and the exponential decay rate is ~0.17, we have A = 100e7° (b) See Figure 1.11, We estimate the halflife by estimating f when the caffeine is reduced by half (so A = 50); this ‘occurs at approximately ¢ = 4 hours. Figure 1.11 e) We want find the value of when 4 = 50: ‘The half-life of caffeine is about 4.077 hours. Ths agrees with what we saw in Figure 39, Since the facor by which the prices have increased aer time tis given by (1.05) the time after which the prices have doubled solves 1.05)! Jog 2 = log(1.05") = Hog(1.05) log? Tog 1.05 14.21 yean, sasowmions 21 440. Given the doubling time of S hours, we can solve forthe bactera's yrowth rates 2Py = Poe™* not So the growth of the bacteria population is i Pree ‘We want 0 find # such that 37> = Poettt2"l8 ‘Therefore we cancel Py and apply In. We get 7.925 hours. 441, In ten years, the substance has decayed to 4056 ofits orginal mas. In another ten years, it will decay by an additional factor of 40%, so the amount remaining after 20 years will be 100 409% - 40% = 16 ke. Using the exponential decay equation P= Pye", we can solve forthe substance’s decay constant ki (Po = 0.37) = Pre“ (0.7) 20 Knowing this deeay constant, we can solve for the halle fusing the formula OP» = Ppetso0s20 20in(0. = 38.87 hours 43, Let B represent the sales (in million of dollars) at Borders bookstores f yeas since 1991. Since B= ‘we want the continuous growth rate, we write B = T8e, We use the information from 1994, that B tofind k: 78 when t = 0 and 12 when f= 3, We have B = 78, which represents an annual (continuous) percent growth rate of 83.54. 444. Te population has neeased hy a factor of $8509 = 1.4 in 10 years. Thus we have te formula P = 40,000,000(1.4)"7 and t/10 gives the number of 10-year periods that have passed since 1980. {In 1980, 2/10 = 0, so we have P = 40.0001,140 in 1990, 1/10 = 1, 80 P = 40.000.000(1:4) = 36,000.000. In 2000, £/10 = 2, so P = 40,000.000(1-4)* = 78,400,000. “To find the doubling time, solve 80,000,000) = 40.000,000(1.4)'/"°, o get ¢ = 20.6 year. 45. number of years since 1980. Then the number of vehicles, Vin milions, atime tis given by FO(L.04)! and the number of people, P, in millions, at time tis given by P= 27101)! 22 Chapter One /SOLLTIONS, ov ‘There isan average of one vehicle per person when ‘Thus, we must solve for ¢ the equation: P 170(1.04y' = 227(1.01)', ‘which implies (2s)! = G00! = BE io) = (ony = 170 ‘Taking logs on both sides, Lo 27 Hog ee tg 2 Tero, log (#35. 106 (FB). 00 years, log (97. So there was, according to this model, about one vehicle per person in 1990, 46, Let n be the infant monlity of Senegal. Asa function of time tn is given by n= no(0.90). “Totind when n = 0.50no (so the numberof eases hasbeen reduced by S05), we solve 0.50 = (0.90)", Jog(0.50) og(0.90), {og(0.50) imtoo) * 8383008 47, (a) We ow the dscay follows the eeation P= Poe. and that 108 ofthe pollaton i ema ater 5 hours (meaning that 90% i ef, Therefore, 0.90P) = Poe™"* += -Lin(o.90) “Thus. after 10 hors Px Pye !4-02310000) P= PUY =081F, $0 816 ofthe origina aout ie. () We want 0 sove forthe ime when P 0.30Ph: = Pret m0.90) (e090) go" 31n(0.50) (0.50) 2.9 hours © P 10 a (@) When highly polluted air is filtered, there is more pollutant per liter of air to remove. Ifa fixed amount of air is cleaned every day, thee is a higher amount of pollutant removed earlier in the process. 1ssowmons 23 48. (a) The pressure P a 6198 meters is given in terms of the pressure Pp at sea level to be P= Pye 20ers ‘Ppel-om123005 Pye B78 SOATSBP) or about 47.5% of sea level pressure (b) AU = 12,000 meters, we have P= Pre Doo0i28 = Pel 200012920 = Pooh 0.28697) or about 23.7% of sea level pressure. 49. Since the amount of strontim-90 remaining halves every 29 years, we ean solve forthe decay constant; 0.5, = Poe 0/2) be 5 Knowing this, we ean look for the time ¢ in which P = 0.10Ps, or O.10P = Poel 1n(0.10) _ Ta(0.3) bs 1 eM) 0.5508 -hart0-te 441.43 years, so the painting is a fake. 21-1074. Thus, & Solutions for Section 1.5 Exercises Py Copter One SOLUTIONS sin is postive cos F ispositve tan 4 is positive 5 is posi sin dr cosdr = -1 isnegative tan3r ‘ssowuTions 25, 25 Chapter One /SOLUTIONS 8. 4 radians radians cos is negative tan4_ is postive 9, —L radian ABBE = — (ME) = 60° sin(-1) is negative cos(-1)_ ispostve tan (-1) Isnegatve 10. > (by piewre) = 0.808, nL sssownons 27 By the Pythagorean Theorem, (cos so (sin §) = 1 - (cos $)?-andsin ¢ = JT (60s 3) = y= (ROOF ~ 0.588 ‘We take the postive square root since by the picture we know that sin § is positive. 0) L 1 on By the Pythagorean Theorem, (cos 7 (cos #5)? = 1~(sin )* andcos /T= (0-259)? = 0.966. We take the positive square root since bythe picture we know that cos 13, (a) We determine the amplitude of y by looking atthe coefficient of the cosine tem. Her. the coeficien is 1, s0 the amplitude of y is 1. Nov thatthe constant term does eo affect the amplitude. (6) We know that the cosine function cos-x repeats itself at x = 2x. so te function cos(3zx) must repeat itself when 3r = 2x, oral x = 2x/3, Sothe period of y is 27/3. Here as well the constant term his no effect, (€) The graph of y is shown inthe figure below. y 5 4 ¥ 14, (a) A(t) = 2eos(t— 7/2) (0) f(t) = 2eost (6) aft) = 2eon(t + 7/2) This raph iain carve wiper an mpd 2.50 gen by J(2) = 2sin (2) period 6x and amplitude 5 s0itis given by f(z) = 30s (2), 16, This graph is a cosine curve wit 17. The graph is an inverted sine curve with amplitude 1 and period 2, sifted up by 2 soitis given by f(z) = 4 sin(22). 18 Ths path an eres cre ith arp ad pero ogy le) 18. Ths pap nie ane cn Pro 5b she by oii ghenby a) = 26a (2) +2 so() 20, THis graph isan inverted cosine curve with amplitude 8 and period 20, soit is given by f(x) = 2. The graph isa sine curve which has been shifled up by 2, s0 f(z) = (sin 2) +2. 22, This graph has period 6, amplitude Sand no vertical or horizontal shifl so itis given by 2) = sun (Zs) M2) = sin ( 28 Chapter One ‘SOLUTIONS. 23, This graph has period 5, amplitude 1 and no venical shift or horizontal shit rom sin x. s0 itis given by Sex) =sin (2) 24, This graph has period 8 amplitude 3. anda vertical tit of 3 with no horizontal shift Is given by Ke 2) =343 ange iE arecos\ 1 recs iv Se mor /2 Sy Sa/2 Figure 1.13, Figure 1.14 (©) The domain =1 & 1 £ 1 while the range is 0 < y < x. Se Figure 113, (4) The domain of areeos and aresin are the sme, “1S # 1 since their inverses (sine and cosine) only take on values in his ange. (The domain of the origina sine function was restricted tothe the interval [~. $] to construc the aresine funtion. Hene, the range of sesne is also [~ §.§]. Now. if we resect the domain of cosine in the same way, we obtain an accosine curve which is nota function (See Figur 1.14) For example, for = 0, y = arecos wil have to talues,~$, and $. Also, it gives no vales for < 0 0 isnot very sel. The domain of cosine should instead be restricted [02] sothaty = arcs. gives a urigue y foreach vale of x Solutions for Section 1.6 Exercises 1. Exponential growth dominates power growth as x + 20, 0 10-2 is large. 2, As2 + 00, 0.252" jg larger than 25.0002~% BASE 0c. y 00. As 00. 9 + ~00. 4. ASE 00,4 0. Aer 20,9 > 0, ‘SSOLUTIONS —3t 8, (1) Degree > 3, leading coefficient negative. (ily Degree > 4, leading coettcient positive. (ll Degree > 4, leading eveficient negative (V) Degree > 5, leading cvetficient negative. (V) Degree > 5, leading coetficent postive. 6. (a) I and II1 because in both eases, the numerator and denominator each have 2” as the highest power, with coefficient Therefore, asx + boo. (b) L since a. boo. () I and Il since replacing x by x leaves the graph of the function unchanged. (@) None {e) IL, since the denominator is zero and f(2) tends to +36 when x = 1 7. (a) From the 2-intercepts, we know the equation has the form y= Re+ 22 (e- 5). Since y = 2 when x = 0, (2-1-8) Trane ne yates ate me-9 f(a) Boca oor cubic has a 04a 2 and double oo st ~2 tas he frm y= He +2 0422-2) when 2 “Thus our equation is Leen y= - H+ 2-2) 9. (a) & polynomial has the same end behavior as its leading team, s0 this polynomial behaves as ~3z globally. Thus we have: J(2) + -casz 4-00. and fz) + ~20882 + $00. (®) Polynomials behave globally as their leading term, so this rational function behaves globally 2s (3z)/(222), oF 3/2. Thus we have: $(2) 43/2382 -4 00, and f(z) 4 3/2as 2 + +00. () We see froma graph of y = e that M2) 0asz4 20, and f(z) + $00 a2 + 400. Problems 10. f(z) = Ke+3)(2—(e=4) ae equal: oterwise one cant M1 f(z) = he(e + 3)(z ~ 4) = R(29 ~ 24 ~ 122), where k-< 0. (= —F if he horizontal and vera sis are equal, ‘otherwise one cant tel how large is.) 12, f(a) = He +z = le e~ ‘etcal scales are equ otherwise one ean tll how large is) #22? —11e-412), where k <0. how large kis.) iF he horizontal and vertical seales K(o! — Ta 4522 + 912 — 30), where & > 0.8 ifthe horizontal and 32 Chapter One ‘SOLUTIONS, 1B, f(x) = be + 2)(2 = 2)4(2 = 8) = R(x" — Te" + Ge? + 282 — 40), where k <0. = ‘otherwise one can’t tell how large kis.) i the seales are equal; 14, Since this rational function has a vertical asymptote at x = 2, one possibility forthe denominator is 2. The function oes through the origin, so the numerator must have a fcior of. Since there is horizontal asymptote ay = 3, the end bbehavior must give this result. One possible funetion with his behavior is ar y 15, Substituting w = 65 and h = 160, we have @ 0.01(63°"*)(160°7*) (b) We substitu ¢ = 1.5 and h = 180 and solve for w: 15 = 0.01w"** ago"), We have 0 = Seay = Since w°?® = w'/*, we take the fourth power of both sides, giving oF (©) Wesubsinue w= 70 and soe fork in tems of 0.01(70°"* )ho7* Since O75 we take the 4/3 power of each side, giving . n= 11265 16, Let us represent the height by h. Since the volume is 7, we have eh Solving for h gives ‘The graph isin Figure 1.15, We are assuming Visa positive constant Figuie 1.15, vesownions 33 17. (a) (i) The water that has flowed out ofthe pipe in I second isa cylinder of radius rand length 3 em. lis volume is ver (i) the rave of flow isk emvsee instead of 3 cnvsec, the volume is given by h Fk (b) i) The graph of V asa function ofr isa quadratic. See Figure 1.16. Figure 1.16 Figure 1.17 (ii) The graph of V as. function off is line. See Figure 1.17 18, The water which has come through the pipe in | second isa box with sides 5, x, and 2. Its volume is Vesr" 19, The volume of water which has come through the pipe in | second isa cylinder of volume Var “The volume of water which has come through the pipe in second is box with sides 5, 3, and. is volume is ¥ 5-3-2 = loz, 21. Let D(v) be the stopping distance required by an Alpha Romeo aa function of its velocity. The assumption that stopping Gistance is proportional tothe square of velocity is equivalent to the equation Div) = bo hy’ ‘whore kis a constant of proportionality. To determine the value off, we use the fact that (70) = 17, (70) = k( 70)" = 177, Thos, Ie follows that and ‘Thus, at half the speed it requires one fourth the distance, whercas at twice the speed it requires four times the distance, as we would expect from the equation (We could infact have figured it out that way. without solving fork explicitly) (a) Since the rae 7 varies diceetly with the fourth power ofthe radius r, we have the formula Rake where fis a constant 34 Chapter One /SOLUTIONS (b) Given f= 400 forr = 3, we can determine the constuntk. 400 = (3) 400 = &¢81) 400 ka x 1038 So the formula is R= 4.938" © ds Evaluating the formula above at r = 3 R= 4.928(5)" = 3086.42% 23, (a) The object stant at ¢ = 0, when s = r9(0) ~ g(0)2/2 = 0. Thusit starts on the ground, with ero height. (b) The object hits the ground when s = 0. This is satisfied att = 0, before ithas lef the ground, and at some later time “that we must solve for 0 = vot — gf? /2 = (v0 ~ 9t/2) Thus « = O when = Oand when vo ~ gt/2 0/9. The staning time ground at time t = 20/9. The object reaches its maximum height halfway between when it is released and when it hits the ground, oF at (2v0/9)/2 (2) Since we know the time at which the object reaches its maximum height, to find the eight it actually reaches we just use the given Formula, which tells us « at any given f, Substituting ¢ = t/a, shen (0,0 it must hit the te 0/9. 24, The pomegranate is at ground level when f(Q) = ~1612 + 64t = ~161(t ~ 4) = 0,s0 when t = ort = 4 At time £ = 0 itis thrown, so it must hi the ground at € = seconds. The symmetry of is path with respect 10 time may eonvnee you tha teaches its maximum Neigh after 2 seconds. Altematvely, we can think ofthe graph of JO) = ~16t2 + 641 = ~16(t — 2)? + 64 which i a downward parabola with vertex (.. highest point) at (2, 64). The maximum height i #(2) = 64 feet. 28, (a) «5 HF(1, 1) som the graph, we know that 1=a(l)' +01) +e=a+b4e. (id) 16(1,1) isthe vertex, then the axis of symmetry is. = 1, $0 -pen and thus . $.50d But to be the vertex, (1,1) musta be on the graph, so we know that a +b + ¢ = 1, Substituting b = —2a, +a. a(0") + (0) Fe = c.s0€ 3. From (ayii),a = e= 180.0 we get —u +e = 1. which we can rewrite asa = ¢—1,ore= (il) For (0.6) tobe on the graph, we must have (0) = 6. But f(0) (b) To satisfy all the conditions, we must first, from (ai), have & (aX), b= ~2a, so b = =10, Thus the completed equation is f(a) y 5x? —10r +6, whieh satistie all the given conditions. esoLuTIONS — 35 26, At) cannot be of the Form et® or Kt? since (1.0 the ratio of successive values of h is approximately 1.5. Therefore h(t) 9(1.0) = 3.00. However, (2.0) = 24.00 ¢ 3-2°. Therefore y(t) a(t) = 30. Thus f(0) = ef? and since (2.0) = 4.40, we have f() 27, Looking at g, we see that the ratio of the values is: 312 _ BT 49 9.39 aa * Tao é “Thus g is an exponential function, and so f and k are the power functions. Each is of the form az? or az*, (1.0) = 9.01 wo see that fork, the constant coefticient is 9.01. Trial and error gives A(z) = 9.010 43.61 = 9.01(4.84) = 9,01(2.2)?. Thus f(z) = na and we find aby noting that f(9) = 7.29 = 0(9") 2.04. Therefore h(t) must be the exponential, and we see that 2.04(1.5)'. If g(t) = cf?, then © KP, and using 9(.0) = 300, we obiain ia and f(x) = 0.012" 28, The function is a cubic polynomial with postive leading coefficient. Since the igure given inthe text shovs thatthe function turns around once, we know that the function has the shape shown in Figure 1.18. The funetion is below the 5 inthe given graph, and we know that it goes to ++2c as x —> +00 because the leading coeficient is 3. seas for # positive, Therefore, there are exactly three zeros, Two 2210s are shown, and oceur st approximately x = —1 and. ‘The third zero must be tothe right of z = 10/and so oceurs for some x > 10, Figure 1.18 29, Consider the end behavior ofthe graphs that is, as x —+ +90 and x —> ~2c. The ends ofa degree $ polynomial are in ‘Quadrants {and IT if the leading coefficiom is positive or in Quadeants Il and IV ifthe leading coefficient i ne ‘Thus, there mus beat least one roo. Since the degree is 5. there ean be no more thin roots. Thus, there may be 1. 4, oF 5 roots. Graphs showing these five possibilities are shown in Figure 1.19. Sr estas (@20008 (to Figure 1.19 30. (a) Since as De and ay = ~€.S0, (2) = (e+ 2c)? and g(x) = (x ~ e)*. Then we have Je) (e+ 20° tert, der +46 396 Chapter One ‘SOLUTIONS (b) Since © > 0. the graph of f(a) is the parabola y = 0. Notice that the graph is symmetric about the y-axis and lims-soe f(2) = 2. 32, (@) Ith mv @1 @ 0 33, The graphs of both these functions will semble that of 2* ona are enough window, One way to tackle the problem sto raph them bot along with 2° if you like) i successively larger windows unt the graphs come together. n Figure 1.21, 4.9 and 2" are graphed in four windows. Inthe larges! ofthe four windows the graphs are indistinguishable, es required. ‘Answers may vary. ¥ oo + wo? too 10° Figure 1.21 ‘SSOLUTIONS 37 3M. The graphs are shown in Figure 1.22. @oy ooo# ® y ob atnise est a sn 5 " Figure 1.22 (2) a(x) = A(ENGINE - WIND) = (Fe ~ hu®), where bis a positive constant (®) A possible graph is in Figure 1.23. Figure 1.23, 36. Suppose the length is land the side ofthe square end is. Since the int i the perimeter ofthe square cross-section, we Know Length + Girth = 1 + 4s 08. In addition, the volume is given by 1, so substituting for? = 108 — 4s, we have Vy = 6?(108 ~ 45) = 1080? ~ 4s". ‘The graph isin Figure 1.24 7 Figure 1.24 38 Chapter One /SOLUTIONS Solutions for Section 1.7 Exercises 1. Yes, because 2x + 2? js defined forall x 2. No, because 2x + 27? is undtined at: = 3. Yes, because 2 ~ 2s not zero on this interval 4. No, because 2 ~ 2 2 Yes, because 2x — 5 is positive for3 0, we have [2 2,0 f(z) = —1. Thus the function is given by s0 f is not continuous on any interval containing 2 = 0. 15, For any values of k, the function is continuous on any interval that does not contain x ‘Since 5z* — Or? = 52°(x — 2), we cam cancel (z ~ 2) provided x # 2, giving Per Figure 1.28, (b) In order for ft0 approach the horizontal asymptote at 9 from above it is necessary that f eventually become concave up. It is therefore not possible for wo be concave down for all x > 6, 19, The drug first increases linearly for half a second, a the end of which time there is 0.6 min the body. Thus, for 0 < t-< 0.3 the function is linear with slope 0.6/0.8 = 1.2: QH1 for O 0.5, dhe quantity dacays exponentially ata continuous rate of 0.002, so @ has the en" <, Q We choose so that Q = 0.6 when ¢ = 0.5: “Ae 2OPHO) = 4,-0.0. 06022 Lat o Ois any constant, 10, Since this funetion has y-intercept at (0.2), we expeet ito have the form y = 2eM*. Again, we find k by forcing the ‘other point o ie on the graph: Wd) 7 “This value is negative, which makes sense sine the graph shows exponential decay The Rial equation thea, s 0.34657, pa 20 OT 15, This looks like a fourth degree polynomial with roots at ~5 and —1 and a double root at 3. The leading coefficient is negative, and 50 a possible formula is ya (2 +9) +(e - 3) 16. ‘This looks like a rational function. There are venical assmptotes at 2 = —2 and x = 2 and so one possibilty forthe denominator is 2” — 4. There is a horizontal asymprote at y = 3 and so the numerator might be 32?. In addition, y(0) = 0 ‘which isthe case with the numerator of 3z%, A possible formula is 3a w=ae 17, There are many solutions for a graph like this one, The simplest is y = 1 ~e°*, which gives the graph of y = e*, flipped lover the z-axis and moved up by 1. The resulting graph passes through the origin and approaches y = 1 as an upper bound, the two features of the given graph, 18, There is no break inthe graph of f(z) although it does heve a ‘comer’ at x = 0,s0 f(2) i 19, The graph of g(z) is shown below. 1 has a break at x = 0, so g(2) isnot continuous on {1.1} fe) = eb ———— 20, The graph has no breaks and is therefore continuous. See below: Osind SOLUTIONS to Review Problems forChapler One 43, 21, The graph appears to have a vertical asymptote at t = 0,0 f(t) is not continuous on [1.1] 22, f(a) =Inz, g(z) = x*. (Another possibly: f(z) = 32, gz) =Inz) 23, f(z) =z, glz) = Inz. Problems near equation 5 = b+ mit, To find the slope m, we use 24, (a) We find the slope m and intercept 3S _ 66-113 > 0-0 0.94. 118, so the intercept b is 113, The linear formula is When t = 0, we have $= 113-094. (b) We use the formula $= 113 ~ 0.94", When $ = 20, we have 20 = 113 ~ 0.94¢ and vo ¢ = 98.9. IF this linear model were carec, the average male sperm count would drop below the fentlity level during the year 2038, 25, We will et amount of fuel for take-off. amount of ful fo landing, ‘amount of fuel per mite inthe air, ‘he Fength of the trip in miles. ‘Then Q, the total amount of fuel needed, is given by Qn) =T +L 4 Pm. 26, To find a half-life, we want to find at what ¢ value Q = }Qo. Plugging this into the equation of the decay of plutonium- 240, we have = 222) = Zo.oo0t ‘The only diference in the case of plutoniun-242 is thatthe constant —0.00011 in the exponent is now ~0.0000018, ‘Thus, fllowing the same procedure, the solution for ¢ is 1/2) 385,081 yeas Fo.ano0ors * 385.981 » 6.301 years ' 44 Chapter One ‘SOLUTIONS 227, We can solve for the growth rate of the bacteria using the formula P = Pye: 1500 = s00e"?) i= leso0/son) 6: Knowing the growth ratc, we can find the population P atime P = s00e# = 13.500 bocteria 28 2», Given the doubling time of 2 hours, 20 ove, we ean solve forthe growth rate k using the equation Using the growth rate, we wish to solve for the time ¢ inthe formula P= 100%" where P = 3.200, so 3.200 = 100" f= 10 hour, Assuming the US population grows exponentially, we have 248.7 = 226.5e"* {n(.098) = 0.00936. 10 ‘We want to find the time ¢ in which ‘Thus, the population wll go over 300 million around! the year 2010. Since we are told thatthe rate of decay is continuous, we use the function (Q(t) = Qoe”* to model the decay. where Q(E) is the amount of strontiurs-90 which remains atime f, and Qo isthe original amouat. Then QU) = Goer So after 100 years. Q(100) = Qye 728100 and (100) Qo 0.0846 $0 about 8.46% ofthe strontium-60 remains 31. Ir was the average yearly inflation rae, in decimals, then 4(1 +r) = 2.400,000,s0 r = 211.53, ie.r = 21.1599. 32. The period Te ofthe cant is (by definition!» one year or about 365.24 days, Since the semimajor axis of the earth i 150 million km, we ean use Kepler's Law to derive the constant of proportionality k (Set Te where Se isthe earth's semimajor axis, or 150 million kn 365.24 = 4(130)3 ‘SOLUTIONS to Review Problems for Chapter One 365.24 6 = 0.198, (a0)! [Now that we know the constant of proportionality, we can use it to derive the periods of Mercury and Pluto. For Mercury, Tar = (0-198)(58)? = 87.818 days. For Pluto, Tp = (0.198)(6000)* = 92. 400 days. or (converting Pluto's period 10 yeas). (0.198)(6000)* 5 = 258 yeas 3, (a) Let the height of the ean be A. Then v rh. The surface area consists of the area of the ends (each is =r?) and the curved sides (area 2xrh), so r? + Inch. eS ‘Substituting into the formula for S. we get = rine 100 Figure 1.33, 34. (a) The line given by (0,2) and (1.1) has slope m =1 and y-intercept 2s its equation is yaar. “The points of intersection of this line with the parabola y = 2 are given by Pee-de (2422-1) 242 coordinate of C. When we substitute back into ‘The solution 2 = 1 corresponds to the point we are alrealy given, so the other solution, = ~2, gives the x- er equation to get y, we get the coordinates for C, (= 4), 6 38, Depih 36. 3). 38 Chapter One ‘SOLUTIONS. (t) The line given by (0.0) and (1,1) has slope m: forthe line as we did in par (a) 1 —b,and yi 1 — br +b. We then solve for the points of intersection with y = 2 the same way: #-(1-br-b F4(-r-b (e+ 6r=1) ‘Again, we have the solution a the given point (1,1). and a new solution at x = —b, corresponding tothe other point ‘of intersection C. Substituting back int either equation, we can find the y-coorinate for Cis b%, and thus Cis given bby (~b.b*) This result agrees with te particular case of part (a) where b +1.5sin (Ft) ‘Over the one-year period, the average value is about 75° and the amplitude of the variation is about function assumes its minimum value righ atthe beginning of the year, so we Want a n in yeas, we have the function a (any other answers are possible. depending on how you ead the char.) (a) Yes, fis invenibe, since f is increasing everywhere (by -*(400) isthe year in which 400 million motor vehicles were registered in the world, From the picture, we see that 4° (400) is around 1979. (6) Since the graph of f-" is the reflection of te graph of f over the line y = 2, we get, gure 1.3, ean a (ations) Figure 1.34 Graph of f-* (a) is (2) since itis linear: (b) is f(2) since it has decreasing slope: the slope starts out about 1 and then decreases 19 bout #5 (6) is h(x) since it has increasing slope: the slope stars out about and then increases to about L 39, (a) The period is 2x. 40. (a (b) fier x, the values of cos 20 repeat, but the values of 2 sin 8 do 901 (in Fat, they repeat but tipped over the x-axis). After another that is after a total of 27. the values of eos 26 repeat again, and now the values of 2 sin 6 repeat also, so the function 2sin 6 + 3.cos 28 repeats at that point “The rate is the difference of the rat at which the glucose is being injected, whichis given to be constant, and the rate at which the glucose is being broken down, which is given to be proportional tothe amount of glucose present. ‘Thus we have the Forms Rak-0G ‘where is the rate that the glucose is being injected, a isthe constant relating the rate that i is broken down to the mount present, and G isthe amount present. SOLUTIONS to Review Problems for Chaplet One 47 o AL. (@) r(p) = kp(A ~ p)s where k > isa constan. ©) p= A/2 42, (a) The domain is (0, 4000), the range is (0.10). (b) The domain is (0. 3000), the range is (0. 10"). e) The domain i (0,0.2), the range is (0.0.04). CAS Challenge Problems AB. (a) ACAS gives f(z) = (x ~ a)(z + al( +O ~ (b) The graph of f(z) crosses the z-axis at z 2 = =b, x = crit erosses the y-axis at abe, Since the coefficient of x* (namely 1) is positive, the graph of f looks like that shown in Figure 1.3. y Figure 1.35: Graph of f(z (2 al(z+a)(z+ 02-0) He (@) ACAS gives f(z) = -(z ~ "(2 - 3). (b) For large 2, the graph of f(z) looks lke the graph of y = —2*, so f(z) -+ co as x -+ ~oo and f(z) -+ ~20 a8 2 > 00. The answer o pat (a) shows that f fas a double root at x = 1, s0 near x = 1, the graph of f looks like 4 parabola touching the z-axis at z = 1. Similarly, f has a tiple root at x = 3. Near x = 3, the graph of f looks like the graph of y = 3°, ipped over the z-ais ad shifted tothe right by 3 so tat the “sea is at 2 = 3, See Figure 1.36. Figure 1.8: Gi fle) = -@- (2-3 48 Chapter One ‘SOLUTIONS 48) ) 46. 4a) o a7. a oy ©) w [As 2 + 3c. the terme dominates and tends to 2c. Thus, f(z) -¥ 20 a8 3 + 26. ‘Ast + —3e. the terms ofthe form e**, where k= 6.5.4.3, 2,1 all tend to zer0, Thus, f(x) + 1688 2 4 ~>0. ACAS gives Sa) = (oF + 1Nfe* = 2)leF ~ 2Y(e"F + 27 +4). the factors (e* + 1) and (e® ++ 2e* +4) are never zero. The olher factors each lead to a Since ef is always po 2210, 50 there are (0 7670. The zeros are given by mn2 so r=in2. Thus, one zero is wvice the size ofthe other. Since f(z) = 2° — 1, J) = FN ~ fla) = (@* ~ a) = 2 ar a Using the CAS to atin the function f(2)- and then asking io expand f(J((2))). we get. {Efla)) = 248 422 = 524 De pe a a, “Te degree of f(J(2)) that i, fF composed with ill 2 vines) is 4 = 22. The depos of f(4(J(2))) (hats. composed with itself 3 limes), is 8 = 2°. Each time you substitute f into itself, the degree is multiplied by 2, because sour stig ina dee pola Sowsexpest be dee of SUUUUJULe))) a fc ‘With itself 6 mes) to be G4 = 2° ACAS or division gives 7 f)= =f eareo- 3 Fo8 3 ‘of plz) are much smaller than the values of r(2)/a(2). so ple) = 2° + 3x +9. and rz) = -3, and gl} The vertical asympote is. = 3, Near + = 3, the valu Thus foy= 35 tore car For large. the values of p(2) are much larger than the value of (2) g(x). Thus FE)S IPED S24 00,24 00. Figure 1.37 shows f(x) and y = ~3/(2 ~ 3) for = near 3. Figure 1.38 shows f(z) and y = 2? + 32 +9 for =20 €£$ 20. Note that in each ease the graphs of {and the approximating function are close. Je) 50 Figure 1.37 Figure 1.38: Far-away view of f(2) and yar +art9 ‘CHECK YOUR UNDERSTANDING 49 48, Using the trigonometric expansion capabilities of your CAS, you get something like sin(Sz) = Seos'(x) sin(2) ~ 10cos"(2)sin*(x) + sin"(2). Answers may vary. To get rid of the powers of cosine, use the idemity eos?() sin (x). This gives sin(z) = 5 sin(x) (1 ~ sin’(z))" — 10sin%(x) (1 ~ sin*(2)) + sin"Cz). Finally, using the CAS to simplify. siu(9#) = 5 sin(x) ~20sin%(2) + 16sin®(e). 49. Using the wigonometvic expansion capabilities of your computer algebra yystem. you get something like c05(42) = cos" (2) — Geos" (x)sin"(2) + sin'(r) Answers may vary (a) To got rid of the powers of cosine, use the identity cos*(r) = = sin (2). This ives cos( 42) = cos! (2) = 6.c0s%x) (1 — €057(2)) + (I~ c0s(ai)’ Finally, using the CAS to simplify cas(4z) = 1 ~ 8¢05%(r) + 8cos"(2) (b) This ime we use sin®(2) = 1 — cos*() to get rid of powers of sine, We get 41 = sin®(e))* — Gsin®(x) (1 —sin*(z)) + sin*(x} cos(42) = Ssin’(2) + 8sin'(x). CHECK YOUR UNDERSTANDING 1. False, A line can be put through any two points in the plane. However, if dhe line is vertical i 6 aot the graph of @ function. True. The graph of y = 10" is moved horizontally by h unis if we replace « by # ~ J for some number h. Writing 100 = 10", we have f(z) = 100(10*) = 102-10" = 10°*?. The graph of f(x) = 10° isthe graph of gz) = 10° shied wo units tothe Tet 3 Truc as seen fom the graph 4. False. since og(e — 1) = if — 1 = 10,s0.2 = 11. 5, Truc, The highest degree term in a polynomial determines how the polynomial behaves when 2 is very large inthe postive or negative direction. When n is odd, 2" is postive when is lrge and positive but negative when 1 is large and negative. ‘Thus if polynomial p(z) has odd degre, it will be positive for some values of x and negative for other values of. Since every polynomial is continuous, the Iatermediate Value Theorem then guarantees that p(x) = 0 for some value of 2 6, False, The y-interceptis y = 2+3e™° = 5, Ber 45, 8. True, Suppose we star at r = 23 and increase 2 by 1 unt tory + 1-Ify = b+ mz. the comesponding values of y are b+ may and b + m(xy + 1). Thus y increases by 1 True, since, ast 4 2, we know e~** 4 0, so Aya b+ me +1) = b+ may) =m 9, False. Suppose y = 8. Then increasing x by | increases y by a factor of 5, However increusing 2 by 2 increases y by a factor of 25, not 10, since (Other examples are possible.) 50 Chapter One ‘SOLUTIONS 10, Tine. Suppose y = AD? and we start at the point (x3, .),50 yy = AB", Then inereasing 2y by 1 gives 2) + 1,50 the new y-value, yo is given by AF = Ab = (ABD, ates Thus, has increased by a factor of b, sob = 3, and the function is y = AB. However, ify is inereased by 2, giving x1 + 2. then the new y-value, ysis given by a = ASH = 437137 = 9.43" = Oy, Thus, y has increased by a factor of 9. LL, False, since cos 8 is decreasing and sin is increasing. 12, False. The period is 2 (0.052) = 40 13, ‘True, The period is 2=/(200x) = 1/100 seconds. Thus, the function exceutes 100 cycles in 1 second. 14, False. 118 = =/2.35/2,Sm/2....then 8 — =/2 = 0, 7.27. ...and the tangent is defined (it is 2r0) at these values False. For example, sin(0) # sin((2=)*), since sin(U) = 0 but sin(( 16, Truc. Since sin(9 + 2) = sin forall 8. we have g(#-+ 2n) = e**l6!™) = 68° = 4) forall 8 ra 18, Pale. The inverse function is y = 107 For 2 <0,as increases, 2? decreases, so e~*" increases 19, Truc. If f is increasing then its reflection about the line y = is also increasing. An example is shown in Figure 1.39. The statement is tue, B ‘Troe, sinee | sin(—2)] = | — sin] = sin, 2. True. IY > 1, then ab? + as z > 2.10 g(b), since g is decreasing, so ~a(a) < ~9(b). Therefore fw < then f(a) — g(a) < (0) ~ (). hich means that f(2) + 9(2) i increasing. Let f(x) = e and let g(2) = e7?*. Note fis increasing snc itis an exponential growth function, andy is decreasing sinc itis en exponential decay function, Then f(2)g(z) = e~*. which is decreasing wereases, g(x) decr increasing function increases as is variable i eases As o(z) decreases. so does f(g(2)) because f i increasing an ass, it decreases as its variable deerescs) 5. Fale, For example, f(2) = 2/(z* + 1) has no venical asymptote since the denominator is never 0 J. False. For example, let f(2) log.r. Then f(x) is increasing on [2,2], but f(z) is concave doven, (Other examples are possible.) 3. False. For example, let y = + 1. Then the points (1.2) and (2.3) are on the lin. However the ratios 2 3 and S15 7 are different The ratio y/ is constant for linear functions ofthe form y = mar, bul not in general (Other examples are possible.) . True. For example, f(2) = (0.5) is an exponential function which decreases. (Other examples are possi Fale. For example, ify = 4+ 1 (so m = 4) and x = 1 then y = 5 Inexeasing x by 2 units gives (9) 415 13, Thus, y as increased by 8 units not +2 = 6. (Other examples are possible.) pee = S but i isnot comtinsous at False. For example, let f(z) { 3 TES -tnen (2) is dened at = 3 but is not continuous at 2 = 3, (Other examples are posible) “The constant function (2) = (isthe only function thats both even and odd, This follows, since if ff both even and ‘odd, then, for all 2, f(—2) = f(z) (if f is even) and f(-z) = —f(z) (if f is odd). Thus, for all x, f(z) f(z) ie. (2) = 0, forall. So f(2) = 0 is both even snd odd and isthe ony such function. (a) This statement follows: i we interchange the roles of f and g in the orginal satmeat, we et this statement (b) This statement stu, but it doesnot follow directly from the origina statement, which says nothing about the ease ga) = 0. (6) This follows, since if (a) # Oe original statement would imply f/s is cominuous atx = a, but we are tld iis (a) This does nt follow. Given that fi continuous a x = a and g(a) #0, then the orginal atement says g continuous implies /g continuous, tthe ther way aroun. Infact statement () snot tac f(z) = O forall then g could be any discontinuous, non-ero function and f/g would be zero. and therefore continuous. Thus the condone of the statement would be satisfied, but nt the consi, 52 Chapter One ‘SOLUTIONS PROJECTS FOR CHAPTER ONE 1. Notice that whenever x increases by 0.5. f(2) increases by 1, indicating that f(x) is linear. By see that f(2) = 2x, Similarly, (2) decreases by 1 each time x increases by 0.5. We know, therefore. that g(r) is a Tinear function with slope 54 = ~2. The y-intercept is 10.0 g(z) = 10 — 2r. ‘n(2) isan even function which is always positive. Comparing the values of and A). it appears that Nz) = 2° F(z) is an odd function that seems to vary between =1 and |. We guess that F(z) = sin x and check with a caleulator. G(z) is also an odd function that varies between ~1 and 1. Notice that G(z) = F(22), and thus G(r) = sin 2r ‘Notice also that H(z) is exactly 2 more than F(z) forall x, so H (2) = 2+ sing. \spection, we 2. (a) Compounding daily (continuously), P= Pret $.450,000e40.080218) $1,597 x 10! ‘This amounts to approximately $160 billion. (b) Compounding yearly, 450,000 (1 + 0.06)" 450,000(1.06)%° = $450,000(245585.29) $1.10499882 x 10! This is only about $110.5 billion (e) We first wish to find the interest that will accrue during 1990. For 1990, the principal is $1,105 x 10", At 6% annual interest, during 1990 the money will eam 0.06 x $1.105 x 10? = $6.63 x 10°. The number of seconds in a year is days) (,,houss) (mins) /., sees 502) (4 ins) (gg eee (28523) @°SP) (ie) (ie Thus, over 1990. interest is accumulating at the rate of 31,536,000 sec. $6.63 x 10" 20:68 10. 5210.24 hee 31,536,000 see chet 2ISOLUTIONS 53, CHAPTER TWO Solutions for Section 2.1 Exercises 1. For f between 2 and 5, we have As _ 400-135 _ 265 Average velociy = $F = SSO = FP kote \elociy on this part ofthe trip was 265/3 km, 2 (a) Lets = flO. (i) We wish to find the average velocity between Land ¢ = 1.1, We have average vloiy = L03)= 10 369-3 = 6.3 tes Gi Webave te erage velocity = £000 = JC) _ 0603-3 _ we average velociy = L000 = £1) _ 3.0008 — 9 = 6.05 me Gi) We hve £4001) — f(2) _ 3.006008 — 3 average veloiy = £0:000)= f 1008 = 8 = 6.009 mice (b) We see in part (a) that as we choose a smaller and smaller interval around t = 1 the average velocity appears to be setting closer and closer 106, so we estimate the instantaneous velocity at ¢ = 1 to be G mse. Slope Point 3[-1o Rp > clels [alo 4. The slope is positive at and D: negative at C and F. The slope is most postive at A most negative at F. 5. Using h = 0.1, 0.01, 0.001, we see (3401) -27 _, aT 27.91 (+0018 -27 _ at 27.09 (340.001)? - “0.000 “These calculations suggest that Kim 6, Using radians, Tor] ‘0.901 10001 These values suggest that im 54 (Ohapter Two /SOLUTIONS 7. Using h = 0.1. 0.01, 0.001, we see This suggests that fin, 8, Using h = 0.1, 0.01, 0.001, we see ae or] 2739 ooo | 27196 ooo | 27194 ‘These values suggest that fim, = 2.7 fn fact this limit is. sin(2) vieure sin 2) S228) sown in Figure 2.1, Therefore, fim S29) a, 9, For ~0.3 $803.05 y <3.the graph of y Figure 24 10, For-1 0 <1, 1 Figure 22 1 For-20" <0 £4070 < y 008 te per = 88 hon’ Fgue 23-Thefoe,bytcnglng e 808 _ yorras curve, we see that in degrees, 2ISOLUTIONS 55 1 Fr -05 6508.06 9 S05. gph ty = fag sons iF 24 Ths yg ge curve, we see that inp 6 i Tan GB) = 0.3383. Problems. 2B. sure rs sisaes —— soe 18. sane — ne 16, 0 < slope at C < slope at B < slope of AB < 1 < slope at A. (Note thatthe line y = 4, has slope 1.) " 18, PAN taee nc roses ecu stp mrs 010 peneyer go! yeni = A plenit Between 1804 and 1927, the population ofthe world increased at an average rate of 15.47 people per minute. Similarly, we find the following: Between 1927 and 1960, the increase was 57.68 people per minut. Between 1960 and 1974. the increase was 135.90 people per minute Between 1974 and 1987, the increase was 146.35 people per minute [Between 1987 and 1999, the increase was 158.55 people per minute Since f(t) is concave down between ¢ = 1 and £ = 3, the average velocity between the two times should be less than the instantancous velocity a f= 1 but greater than the instantaneous velocity at ime f = 3.50 D < A < C. Por analogous reasons, F< B < E. Finally, noe that fis deerewsing at = 5 so F < 0, hut increasing tf = 0.50 D > 0. Therefore, the ordering from smallest to greatest ofthe given quantities is PeB 0, there isa 5 > 0 such that a(t) ~ K] 0 (as small as we want, there isa 4 > 0 (sufficiently small) such that ie < 2 < e+ 6 then Mlay~ Ll 0 (as small as we wand, there is a 6 > 0 (sufciently small) such that Pe = 3 < at < c, then We) L 0 (as small as we want), there is a A We) L Land x approaches 1, then p(x) = $5. fz < 1 and 2 approaches 1, then p(z) ‘that plz) approaches asx approaches 1, s0 we say that lim p(r) does not exis > 0 (sufficiently large) such that if x > Ny then 34. Thee is not a single number 2», of h approaching, but not equal 10, zero. Ifwe let h = 0.01, 0.001, 0.0001, 0.00001, we calculate the values (69, 2.7181, and 2.7183. If we let = ~0.01 ~0,001, ~0.0001, 0.00001, we get values 2.7320, 2.7196, 2.7184, and 2.7183. These numbers suggest tht the limit isthe number € = 2.71828... However, these calculations ‘eat ‘ell us thatthe limit is exact e; for that a proof is needed. 30, Divide numerator and denominator by 2 E43 _143/2 $= 95 pe? din Je) = fin EMT fe te. 33. Divide numerator and denominator by z°, giving _ 2a 162 _ 2- 16/2 J@)= aya az+3 ee og ENG _ lite soo(2— 16/2) _ 2 Ba SO) = Nn feed Titteoao(iE +3) ‘3M. Divide numerator and denominator by 2°, giving fey = Ttbe _ Met spe meroolI/r + 3/2") _ 0 Tingle +2) 2 3. Divide numerator and denominator by e*, giving ede? ane M0) = Fere3 * T4387 limesa(+2e"*) _ 3 Timea (2 + 3e7*) hia 12) 22S0LUTONS 63 dete Timgroo(2e% +3) _ 3 36 1) = ary MI) = Tine +2) 2 37, Because the denominator equals O when z = 4, so mus the numerator. This means 4? = 16 and the choices for k are 4 ora. [38 Because the denominator equals 0 when z = 1, so must the numerator, So 1 ~ k +4 = 0. The only possible value of iss, 39, Because the denominator equals 0 when x = —2, so must the numerator. So 4 ~ 8 + of kis Division of numerator and denominator by 2” yields = Wand the only possible value B4Sr+5 143/z+5/2? wrist Perle [As 2 + 20, the limit of the numerator is 1. The limit of the denominator depends upon k. If k > 2, the denominator approaches 20 a8 2 —+ 06,0 the limit of the quotient is 0. If k = 2, the denominator approaches 1 as x > cc, so the limit of the quotient is 1 If < 2 the denominator approaches 0* as 2 —+ oo, so the limit ofthe quotient is 50. Therefore the values of k we are looking for are k > 2. 441, For the mumerator, lim (e ~5) = —5.IfK > 0, lim (e* +3) = 8, so the quotient has a limit of —5/3, tk 0, tim (243) = 4, so the quotient has limit of ~8/4, IF < 0, the limit ofthe quotient is given by lim (e* ~ 5)/(e* +3) 442. By tracing on a calculator or solving equations, we find the following values of 6: Fore =0.2,6 $01. Fore = 0.1, 6 < 0.08. 02, 5 < 0.01. Fore = 0.01. 6 £ 0.008. For = 0.002, < 0.001 Fore = 0.001, 6 < 0.0005. 4B. By tracing on Fore alcuator or sol ‘equations, we find the following values of 8 Fore=0.1,6 $046, Fore = 0.01, 5 < 0.21. For ¢ = 0.001, 5 < 0.1. Thus, we can take 5 < 0.09. “44, The results of Problem 42 suggest that we can choose 6 = e/2. For any € > 0, we want to find the 6 such that Inte) -3] = |-22 43-3 Bal 0 be given. We need to show that we ean choose 5 > 0 so that whenever Jz ~ cl < 6, we will have MCF) + 2) = (Lr + L2)] < First choose 4, > 0 50 that fr ~ of < 6 implies [f(x) ~ La] < $; we ean do this since lim f(2) = Ly. Similarly, choose 6» > 0 9 that x ~ e| 0, there is a 6 > O so that [x ~ c| < 5 implies [f(2)a(2)] <«. fe > 0 is given, choose 5; so that when |z ~ el < 81, we have [f(2)] < vé. This can be done since lime-so f(z) = 0. Similarly, choose By s0 that when [x ~ c| < ds, we have [g(2)| < yé. Then, if we take 6 to be the smaller of dy and 8, we'll have that |2 — e| < 6 implies both [/(z)] < vé and |g(2)] < ve. So when [2 ~ ¢| < 6, we have 1fG2)a(z)] = [Fe lala) < ve > Ye =e. Thus fim f(2}g(=) = 0. (Fz) ~ La) (g(2) ~ Le) + Light) + Laf(2) —"Tiba (zal) — Ligh 1) ~ Lef(2) + LsLa + Lagle) + Laf(e) ~ Lake = fle)gle. (eo tiny (/(#) ~ La) = tn f(e)= tim, Ly = £4 =Zy = 0,usng the second fini prope. Similarly, im (gl) ~ L a Since lim (f(2) ~ L1) = fm (o(2) ~ £2) = 0, we hav that i (4) ~ Lr) (gl) ~ £2) = O by pa (a. From part (b), we have “ ry @ co ae Fe)a(2) = fin ((J(2) ~ La) (a2) ~ La) + Ligh) + Lofle) ~ Lake) 1 (Az) ~ La) gz) ~ La) + li Ee) + Lim bef) + in (LiL) (using limit property 2) + Li Nim gle) + Le lim f(z) ~ Lila (osing limit prope = Lyle + aly ~ be 1 and part () = Libs 2aSOLUTIONS 65 Solutions for Section 2.3 —_—_—_—_—_———— Exercises 1 The derivative, /f(2),is the rate of change of 2 at x = 2. Notice that cach time 2 changes by 0.001 inthe able, the value of changes by 0.012. Therefore, we estimate 12) = faethe Door ‘The fonction values inthe tle look exactly linear because they have been rounded. For example, the exact value of 7° when 2 = 2.001 is 8012006001, not 8.012. Thus. the table can tell us only thatthe derivative is approximately 12, Example 5 on page 82 shows how 10 compute the derivative of f(x) exactly, y 1 = Since sin zis decreasing for values near 2 = 3s, its derivative at 2 = 3 is negative. 3. (a) Using a caleulator we obtain the values found in the table below: 13 Ez tae [7a [18 | herween x = Land 2 £@) = 40) 3-1 (by The average rae of change of f(2 Average rate of change (e) First we find the average rates of change of f(r) = e* between x = 1.9 andr = 2, and between ¢ = 2and r 4) Toble 2.13 = [tt ~ loge [0018 030040 047 (b) The average rate of change of f(z) = log.r between x = Land x = Bis £0) ~ FQ) _ log-log 08 s-1 7 3-1 2 65 Chapter Two ‘SOLUTIONS (€) First we find the average rates of change of f(2) = log x between x = 1.5 and x = 2, and between ¢ = 2 and 23. fog2 ~ log 5 _ 0.90 0.18 2-18 Coie: og2.5 — tog? _ 0.40 ~ 0.30 0.20 25-2 05 Now we approximate the instantaneous rate of change at x = 2 by finding the average of the above rates instantaneous rate sone) O2E4020 _ yyy, th of f(z) = loge tz =2 ) * ltt +h £0) = jn, OF in Evaluating "85245 for = 0.01, 0.01, and 0.0001, we gt 0.43214, 0.49408, 0.43427 50 f'(1) = 049427. The covegnng ca linc tng per rcs te pupa logs emer dor, Hex ae tne un Oc307" fm amar egal vase heist ore. Wevan a ses fom Be Eph oon Heat) > for h < . Le 6. We estimate /"(2) using the average rate of change formula on a small interval around 2. We use the interval 2 2.001. (Any small interval around 2 gives a reasonable answer.) We have £(2.001) ~ f(2) _ $2" —3" _ 9.009899 _ 9 4g a £2) 7. Since f'(2) = 0 where the graph is horizontal, f'(2 The derivative is positive at points b and ¢, but the raph is steeper at x = c, Thus f"(z) = 0.5 atx 2 at x =e, Finally, the derivative is negative at points «and e but the graph is steeper at x = e. Thus, f(x) = ~0.5 at x = a and f(z) = ~2 at x =e. See Table 2.14. Thus, we have f'(d) = 0. 4'(6) = 0.5, #"(e) = 2. f(a) = —05. f'(e) = — Table 2.14 8, One possible choi ‘of points i shown below. 23S0LUTIONS 67 9. (a) The average rate of change from z = a 10 4 = bis the slope of the line between the points on the curve with 2 = a and x = Since the curve is concave down, the line from x = 1 10 x = 3 has a greater slope than the line from 3, and so the average rate of change between x = 1 and x = 3 s greater than that between x = 3 and (b) Since fis increasing, (5) isthe greater. (6) As in par (a), fis concave down and f" is 10, Using the det casing throughout so f(1) is the greater. ion ofthe drvatie, we have 0) = fm S0# N= £00) = tiny 304M)" = 510)* 2 500+ 100 + 3h? = fin my z = finn WOH + 3h oe ee (200 + 5h) ai eg eee ling 100+ 3h = 100, 11, Using the definition ofthe derivative, we have $(-2+h) ~ Hl-2) £2) (8) which goss to 12 ash 0.50 (=2) = 12. 12, Using he defo fhe deraie gol +h) = gl=1) 7 = finn CELA + (1+ 1) = (9(-1)" + 9(-1)) a ce wg 001= 2h HY) = 9-450 i 7 fin DO = 3 45H sy 7 . +30’) = 9-2) = fim ) = fim (1+ 3h) = £0 +1) fC) _ 5, (+h)? #3) = 0 +3) h a) 7 tim LSA 9 +S Sh+ 3h +m mm soa aera oa Jim(o-+ 3h 47°) £0) = im 68 Chapter Two ‘SOLUTIONS. 4 9) = = tim 222M) eo Ha +2 = jim Goh =~} “EGER G 18, g2+h)~ 912) 7h =n? eS DDH ho aN DME mahi? 9'(2) = fim hin tt Teh 16, As we saw in the answer to Problem 10. the slope ofthe tangent line wo f(z) = 52° az = 10 is 100. When 2 = 10, {(£) = 500 so (10.500) is point onthe tangent in. Thus y= 100(2 = 10) + 500 = 100 ~ 500, 17, As we saw in the answer to Problem 11, the slope of the tangent tine to f(z) = 2 at is 12. When 2 J{2) = 8 s0 we know the point (~2. 8) ison the tangent line. Thus the equation ofthe tangent line is y = 12(z + 2)-8= 127 +16 18, We know thatthe slope of the tangent line o f(2) = x when x the tangent line. Thus the equation ofthe tangent ine sy = 1( 20 is 1. When x = 20, f(z) = 20 so (20.20) is on 1, ive of f(2) = Iz? atx = 1. £0+1)=0) _ py te a E-a+hy 1-42 +n) eso A +he AMO EA)? <2-h Meme “Thus the tangent line has a slope of ~2 nd goes through the point (1,1). and so its equation is 20-1) of y= 2043. Problems 20, The statements (100) = 35 and J'(100) = 3 tellus that at x = 100, the valve of the Function is 35 and the function is increasing ata rate of 3 units for a unit increase in x. Since we increase x by 2 units in going from 100 to 12, the value of the function goes up by approximately 2-3 = 6 unit, so 1102) 3542-33546 = 41 21, The coordinates of Aare (4.25). See Figure: ‘The coordinates of B and C are obtained using the slope of the tangent line, Since #"(4) = 1.5, the slope is 1.3 RESOLUTIONS 69 Figure 222 From A to B, Az = 0.2,s0 Sy = 1.5(0.2) = 0.3. Thus, at C we have y = 25 + 0.3 = 25.3, The coordinates of Base (4.2.25.8). From Ato C, Ar = -0.1, so Ay = 1.5(=0.1) = ~0.15. Thus, at C we have y = 25 ~ 0.15 = 24.89. The coordinates of C are (3.9. 24.83). 22, (a) Since the point B = (2.5) ison the graph of g, we have g(2) (b) The slope of the tangent line touching the graph at x = 2 s given by Rise _ 55.02 _ 0.02 = ae to Slope | (a smpe= 73) fe) | ere (a) (4) > £03) since fis increasing. 70 Chapter Two ‘SOLUTIONS (b) From the figure. it appears that f(2) ~ f(1) > £08) ~ 2). LEV=1) represents the slope ofthe scant line connccting the graph atx = 1 and x = 2. This greater than 0a the slope ofthe seat line connecting the graph al = = 1 and = 3 which is £8.= 0) (2) The function is steeper atx = 1 than ate = 490 f'(1) > f°), 25, Figure 2.23 shows the quantities in which we are intresed. | Sone = 12) | See = J") | | te 2 Figure 223, #108) = (2) = #53242. = slope ofthe secant line from f(2) 0 f(3). From Figure 223 iis clear that 0 < f(3) ~ (2) < (2). By estnding te secant Tne past the pont (3, /(8)) we ean see that ities above the tangent line at Thus 0 FCA) /4 n ec 6 Figure 2.25, zasoumions 71 (2) For the line from A to B, 2 Figue 226 28, Using a difference quotient with A = O.00L, say we find 1) we EL in(.001) ~ 11n(1) 5 7 05 ra ee 1.00 1) wy 20001 In(2.001) = 21(2) _ sae oe 1.6934 “The fact that fs larger atx = 2 than as = 1 suggeststat J is concave up on te interval (1,2) 29. (a) hin degrees 0 ad _ sinh Ft = fy 7 To four decimal places sin0.2 _ sina _ sin 001 goras 02 * "Gr * oor ~~ oom 80S s0 (0) = 0.017 (0) Consider the ratio 22, As we approach O, the numerator, sn h wil be much smaller in magnitude if his in degrees than it would be if h were in radians. For example if ft = 1° radian, sin h = O.8415, but if h = 1 degree, sin h = 0.01745. This, since the numerator is smallet fr h measured in degrees while the denominator isthe same. wwe expect the ratio 824 to be smaller. 30, We want f'(2). The exact answer is £Q Jig 22 =f2) fim, 22H but we ean approximate this. If = 0.001, then (2.001) — 4 “por = and if A = 0,0001 then (2.0101)29° 0.0007 778, 30 f'(2) 26.77 31. Notice that we can’t get all the information we want just from the graph of f for 0 < x < 2, shown on the left in Figure 2.27, Looking a this graph, it looks as ifthe slope at 2 = 0 is 0, But if we zoom in on the graph near 2 = 0, we get the graph of f for 0 < x < 0.05, shown on the right in Figure 2.27. We see tha f does dip down quite abit berween = O and x = 0.11. In fact, it now looks like J°(0) is around —1. Note that since f(z) is undefined for x < 0, this derivative only makes sense as we approach zero from the right. 72 Chapter Two /SOLUTIONS y anor A 067 062 0.83 0.08 0.05" 0.008 0.0075 0. 0.0125 0015 0017 Figure 227 \We room in on the graph of f near x = 1 to get more accurate picture from which 1 estimate f"(1). A graph of {f for 0.7 Sx < 1,3 is shown in Figure 2.28. (Keep in mind that the axes shown in this graph don't cross atthe origin!) Here we see that °(1) = 3.5. Ma)= 52? —2 oroso9 | iizts Fv 228 * (+h) =f) Infcos(1 + h)) = In(cos t) 10) = fin EARL fg eee For hi = 0.001. the difference quotient = ~1.55912; for = 0.0001, the difference quotient = 160Us Fate of change of f therefore appears to be about ~ 1.588 at r 0.0001. then Infoos($ + 0.0001)] ~ In(cos $) ooo. ‘The instantancous rate of change of f appears to be about —1 at 33, We want co approximate P* (i) and P"(2). Since for small PU) ~ PCO) a difference quotient POs 1.15(.014)°°" — 118 PO)= ao 0.01509 billion year = 16.0 million poople/year 1.15(1.014)" 2! —1.150.014)" Tor 10164 billion /year 16:4 million people/vear zasowmions 73, Ser (a) From the figure above. it appears that the slopes of the tangent lines to the two graphs are the same at each 2. For .£ = Oy the slopes of the tangents to the graphs of f(2) and g(x) at O are Fors = {0+ h)~ FO) 0+ h) ~ 910) i A £(0) = fen, 9(0) = fim, ahh) ~ 9(0) = Jim 2 the slopes of the tangents tothe graphs of f(+) and g(x) are sim, LEER = $2) . vey tg Ha Fo eal 10) fn, OA) wn FO HA 40" win OEMS ae “Es 7 _ ig, taht = pean Ht oT a thee fin 2EREE BOA) a amt ae 1 sia, (2+ 34) = fin 2 is =2 For 2 = 20, the slopes ofthe tangents tothe graphs of f(r) and g(2) are 74 Chapter wo /SOLUTIONS Hz +h) ~ fl20) glzo+h) ~ (zo) F060) = jin, HEE) G20) jin tg $l Hak tig EOS (f(z) 43) mee a tag $leb + 20h tg eI = flea = 7 pee wig, AE og, Mobb Beh 08) = bd i ts, 7 1 oh + $h? sot og, AH 3 (2+ 3h) = fim : = hy (+54) ‘(r) = gz +h) — gfz) 2) = fy EO =, fet N40 -Ule) +0) fs 7 = jim = F(a) 3. As h gets smaller, round-off error becomes important. When ht = 10", the quantity 2* ~ 1 is so close to O thatthe ‘calculator rounds of the difference 100, making the difference quotient 0. The same thing will happen when hk = 1072 He +h) = (2) 7h Solutions for Section 2. Exercises 1. The graph is that ofthe line y = ~2z +2. The slope. and hence the derivative, is —2 2ASOLUTONS — 75 4. The slope of this curve is approximately ~1 at 2 = —4 and at 2 = 4, appr approximately 1 at r = 0. fly Oat 2 = =2.5 and x % 10, a Chapter Two ‘SOLUTIONS = [me = [mr = [er = [ez 998 | 0.0030 1998 [0.6027 5958 | 1.6000 Soom [23028 10.999 | -0.0010 1.999 | 0.6926, 499 | 1.002 9990 | 23025 1.000 } 0.0000 2.000 | 0.6831 5.000 | 1.6053 ooo | 2.3026 1.001 | .0010 2.001 | 0.6536 3.001 | 1.6096 0001 | 23027 1.002 | 0.0020 2002 | o.6eai S002 | 1.6058 10002 | 23028 [Atz = 1, the values of ux are increasing by 0.001 for each increase in z of 0.001, 50 the derivative appears to be 1 ‘At.z = 2, the increase is 0.0005 for each increase of 0.001, so the derivative appears to be 0.5. Atz = 3, by 0.0002 foreach increase of 0.001 in x. 50 the derivative appears to be 0.2. And at x = 10, the nerease is 0.0001 over intervals of 0.001, so the derivative appears tobe 0.1. These valves suggest an inverse relationship between x and f'(z)- namely f'(z) = £ (a) We use the interval (othe right of x = 2to estimate the derivative. (Alternately, we could use the interval tothe left fof 2, oF we could use both and average the results.) We have HOES] Laz We estimate f'(2) = 3. (b) We know that #"(2) is positive when f(x) is increasing and negative when f(2) is decreasing, soit appears that J (2) is positive for 0 < x <4 and is negative for 4 4.2. Ay _ 14-60 wc atante By 08 9g berween s = 47 and By _ 90-74 2 between = Sand c= 57 Ar ~ 57-5. E Thus (2) shoul be clos to 3 near = 5.2. n ; aa 24, This is a line with slope —2, so the derivative is the constant function "(r) y = ~2. See Figure 2.28 2. The graph is a horizontal line at 2 yay Figure 229 25, This function is decreasing for x < 2 and increasing for > 2 and so the derivative is negative for x < 2 and positive for x > 2. One possible graph is shown in Figure 2.30. y 12) TAT * Figure 230 ~ ee fe) “This function is increasing for approximately x < 1 and x > 4.5 und is deereasing for approximately 1 < x < 4.5. The derivative is postive for 2 < 1 and x > 4.5 and negative for 1 < x < 4.5. One possible graph is shown in Figure 2.31. \ se) af Figure 2.31 80 Chapter Two /SOLUTIONS a re) 0 30, This function is inereasing for x < 1 and is decreasing for x > 1 so the derivative is postive for x < 1 and negative for > I. Imaddition, as x ges large, the graph of f(x) gets more and more horizontal. Thus, as x gets lage, f"(x) gets closer and closer to 0, One possible graph is shown in Figure 2.32. 3 sa) =2and increasing for values of z less than ~2, i decreasing between From the given information we know that f 2, and is constant for + > 2. Figure 2.33 shows a possible graph—yours may be different. Figure 239 2asoumons at ‘3M. Figure 2.34 shows possible graph ~ yours may be differen. Figure 2.34 38 (za (er (zs Or 36. The derivative is zer0 whenever the graph of the original funtion is horizontal. Since the curren is proportional to the derivative of the vokage, segments where the curren is zero altemate with positive segments where the vollase is Increasing and negative segments where the voltage is decreasing. See Figure 2.35. Nove thatthe derivative does not exist \where the graph has a comer. Figure 2.35, 37. (a) Graph I () Graph | (e) Graph il 3B t=3 Ot et @ 1 -2 38. (a) The population varies periodically witha period of | year. See below. a sree FMAMJJASONDG (b) The population is at a maximum on July 1". At this ime sin(2x¢ — 4000+ 500(1) = 4500. Similarly the population is aa minimum on 0 the minimum popolation is 4000 + 500(—1) = 3500. (©) The rate of change is most postive about April 1" and most negative around October 1° (@) Since the population isa its maximum around July 1, its rate of change is about O then, 1, s0 the actual maximum population is nuary 1, At this time. sin(2r¢— 2) = —1. 82 Chapter Two /SOLUTIONS 40, (6) The faction fisneemsing where fi postive sforzy <2 1, more should be spent 20, Since 2H=P(8 jam estimate of P66), we may think of P'(66) as an etme of P(GT)— P(G6),and the la umber of people between 66 and 67 inches tll, Aleratively, since 2S,8-=F188) isa peuer estimate of F7(66), we may regard P"(66) as an estimate of the numberof people of eight betwen 63.3 and 66.3 inches. The units for P*(2) ae people per inc, Since there were 250 milion people atthe 1990 census, we might guess that there are about 200 million fll-grown persons inthe US whose hight are distributed berseen 64" (3") and 75"(0'3"). Ther re probably Guitea few people of height OO"-perhaps 1§ what you'd expect from an even or uniform, istibuion-beease i's neatly average. An even distribution would yield P”(66) = 222!22 = 13 million per inch-so we can expect P'(66) 10 be perhaps 191.5) = 20. (2) isnever negative becuse P() fs never decreasing. To see this, le’ look at an example involving particular sal of 7 58) 2 = 70. The value P(7O) represents the number of people whose height is less than or equal 10 70 inches, and P(71) represents the numberof people hose height f les than or equal 10 71 inches, Since everyone shat than 70 inches is ls shoner than 71 inches, P(T0) < PCT1). In general, P(z) sO for sl x, and increas and is eventually constant (or large enough 2). iter isthe 21. (a) The units of compliance are units of volume per units of pressure, or liters per centimeter of water. (b) The increase in volume for a 5 em reduction in pressure is largest between 10 and 15 em. Thus, the compliance appears maximum between 10 and 15 em of pressure reduction, The derivative is piven by the slope, so 0.70 ~ 0.9 15-10 Compliance = 0.042 liters per cemimeter. (e) When the lung is nearly full, teannot expand much more to aecommodite more ait Solutions for Section 2.6 —_—<$< Exercises 1. (a) Since the graph i below the z-axis atx = 2, (2) ivneg (b) Since f(z) is decreasing at 2 = 2, f' (2) is negative. (©) Since f(2) is concave up t x = 2, £"(2) is positive. 22, By noting whether f(x) is postive or negative, ineteasing or deereasing, and eoneave up or down at each of the is, we get the completed Table 2.16: Table 2.16 Pom PP 3. At B both dy/dde and d2y/de? are positive because at B the graph is increasing, so dy/d > 0 and concave up. so Py/dz* > 0. 85 Chapter Two ‘SOLUTIONS 4. The velocity isthe derivative ofthe distance, that is. v(¢) = s(t). Therefore, we have ot = fn, i +h)? +3) a) i ei 10th + 5h? SRS he fg MOB = fn oe) = 101 vot The cceraion the dean of vei salt) =e hy — 108 at) = i 1t = 14 di, 1 = km 5. The Function is everywhere increasing and concave up, One possible graph is shown in Figure 2.38, Figure 2.38, 6, The graph must be everywhere decreasing and concave up on some intervals and concave down on other incervals. One possibility is shown in Figure 2.3, Figure 2.99 1, Since velocity is postive and acceleration is negative, we have J > O and J" < 0, and so the paph si concave down. See Figure 240 Figure 2.40 “frre 01280198 4P/eP WAY < Pll gP PUP > IP/eEP (A) 0 < Pld P PUD < ID/P A) “LE be emnbig vay sotous ain Buowr way (1) 4s aes aqqssod Kuo a 81 Zz Fuynous “Fz wap sors 39 as (2) uote “Fz Jo miny Ypeas aM “2 = zu Buy] W>EUEI A Wo; a 5 "E < 7 JO} DAIND BAY BADGE Sq] UE Z BOIS sey ¢ = 2 ve aan oy OF SUN URE a “OSI “IE AIREY HL UNoUS YUE 2p 9 8400] YETI! ON “UNO 249009 St fue (osausop Knuonbosqns Keus f fno4p) @-< z = (6) f Sauls Futseasout SUM Uonouny Ag = 7 Jo WA AN OL “9H z_ e551 De=ZE ~ wonessjaaae a8esa6y One = m 1515020) T= Te = TipeTAN GT STUD ~ uoweojonoe asessay pomoe = + 2asny anti “29sy2asp) 40 ‘puodss Jad 9997} 9m WopEID|SNOR YO SIUM ML sou oq 2090 Aus099 Jo aBueyD Jo 64 erase 241 AIP|NAED 94 “[EZOIUL UR 940 UOMEID|RADE 9BEIDAY Y NSEDU OL “ST “e161 uy seaK sad sie9 wo} gee MOgE JO ‘ies © we Suseosout seas Sp oq UL sna JeRuassed Jo Joqumy ayy IMaK 49d ssE9 UONMU EE = (CLG), f EM 225 344, Ot _ 9461 ~ 0861 ee ~ fos ott (e461), f a¥wIs9 01 (BGT OF OZ6E TeH>1HE 241 WO PFRUIO} 9BuRUD Jo 9H 9BEIDKF 341 96M AML) ‘peak aad Sino owt 1e"tz ~ e'0F) st BUCY. yp Jo 2163 24 381999 aa;sod 34 01 sade ms9p puodos a4, “Buisraiou st swe Jo saquimu 241 souls axmusod oq 01 Eade (1) f “SAEAUDP OU (©) “FL we = (e160) fF FUG 1 9fueys Jo os 9 “9961 PuE OS6I UODNID9 a[IYA “7 Jed SA UOTE ge" Jo ates 241 “961 UE OPEL UBDMIDG “s}duIExD 104 “Buysearou St WS % swaygord o> @),f o> (sa 0> af 0< (fz o<@t o> (hsm o=S o>) 0 OFS (Of % o<,s ONS 8 48 snounos 92 88 Chapter Two /SOLUTIONS 18. (a) wey wsnty (b) Asa function of quantity. utility is inereasing but a a decreasing rate: the graph i the derivative of uilty is positive, but the second desivative of uly is negative creasing but concave down, So 19, Since all advertising campaigns are assumed to produce an increase in sales, a graph of sales against time would be ‘expected to havea positive slope ‘A positive second derivative means the rate at which sales are increasing is increasing. [Fa positive second derivative is observed during a new campaign, itis reasonable to conclude tht this increase inthe rate sales are increasing is caused by the new campaign-which is therefore judged a success. A negative second derivative means a decrease inthe rate at ‘which sales are increasing. and therefore sugyests the new campaign isa failure. 20. ta ae itt -3 “4 hy Essetly one. There can't be mare than one zera because fis increasing everywhere, There does have to be one zero because f slays below is tangent line (dotted line in above graph), and therefore f must cross the z-axis. (6) The equation of the (dotted) tangent line is y = 2 — 4. and soit esses the 2-axis at x = 1. Therelore the zero of| {f must be between 2 = Land 2 = 3. (a) lim. f(2) = —20, because f is increasing and concave down. Thus, as ¢ > ~90, f(2) decreases, at a faster and Faster rate (e) Yes (f) No. The slope is decreasing since f is concave down, so f'(1) > f"(S)sie. f'(1) > 21. (a) The EPA will say thatthe rate of discharge is il rising. The industry will say that the rate of discharge is increasing less quickly, and may soon level off or even start ofall (by The EPA will sy tha the rate at which pollutants are being discharged i levelling off, but not to zero — so pollutants will continue to be dunaped inthe lake. The industry will say thatthe rate of discharge has decreased significantly, 22, Since J" is everywhere positive, fis everywhere increwing. Hence the greatest value of f is at z¢ and the least value of {Fis az. Directly from the graph, we see that fis greatest at 9 and least ato. Since #” gives the slope of the graph fof f'.f" is greatest where fis sing mast rapidly, namely at xg, and is least where J” falling most rapidly, namely 23, tay Bowhere ff" > O) and E (where f', f" <0) (by A where f = f= O)and D (where f= f= 0) Solutions for Section 2.7 —___ Exercises 1. (a) Function f isnot continuous az = (b) Function f appears not differentiable at = 1, 2.3 27SOLUTIONS 89 (2) Function g appears continuous a al -values shown (b) Function g appear ot dillerentabe atx = 2.4. AU = 4.the gr has a comer, 0 the derivative doesnt exist 5 (2) The absolute vale function i continuous everywhere. See Figure 2.42 (b) The abso value Fueton ic not dilferemabe t= 0. The graph asa comer at = 0, which suggests fis nt ditferemiable ther. (See Figure 22.) This sconimed bythe act that he init of the diference quotient Sim LEM) = fla) ay i the curve is vertical, so the derivative does not exist. At does not exis for 2 since the following limit does not ta Figure 242 4 No, there are sharp turning points. 5. Yes Problems 6, We want fok t (ie + 0.0001)" = (0.0001)? rs [Ash -+O fom postive or negative umber, he deren quent approaches . (Try vahatng it for = 0.001, Oooo, ee) Sot appear ther isa derivative a r= and ha t's drat is zero. How can is be iff hs come “Te answer tis in th fat that wha appcars to be a comers in fat soothe you zoom inthe raph of looks lke sight ine wih sips O! Se Figure 2% say Te) o 1 2 “02 “or 0 O12 Figure 243: Closesups of f(2) = (2 + 0.0001)'"* showing differentabilty st r = 0 7. Yes, fis differentiable atx 0, singe its graph does not have a "eorner” at 2 =D. See below. 1.61 80 Chapter Two ‘SOLUTIONS Another way o sce this is by computing: sen LOD= LO) = jy ED? fs aa Since tl? =H? we ave im LL) ° So f is differentiable a0 and f'(0) = 0 8. As we can seein Figure 2.44, f oscillates infinitely often between the z-axis and the line y = 2x near the origin. This ‘means a Tine from (0,0) 1 a point (h. f(A) on the graph off altermates between slope 0 (when f(H) = 0) and stope 2 (when f(s) = 2h) infinitely often as h tends to zero. Therefore, there is no limit of the slope of this ine as tends to zero, and thus there is no derivative atthe origin. Anoher way to see this is by noting that nH OD «yo MD * (on) 1) doesnot exit, since sin( doesnot havea mit tends to zor, Ths, fis ot ifeenible at = 0 Figure 2.44 9, We-can se from Figure 2.45 thatthe graph of f oscillates infinitely often berween the curves y = 2? and the vain. Thus the slope af te Tin om (0.0) (0) osiates between (when f(A) = A? and and —h (when f(h) = —h? and = has tends 0 zr. So the init ofthe slope as hens ozr0 0, which is the derivative of f at the origin, Another way to sce this isto observe that £10) = $0) gy (f sat) i Jin 1 ight) = since fay h = O and ~1 < sin() $ 1 forany h Ths i ilferentable atx = 0 and (0) Figure 245 2TSOLUTIONS — ot 10, (a) The graph is concave up everywhere, except at = where the derivative is undefined. This i the case ifthe graph hhas a comer at z = 2. One possible graph is shown 2 Figure 2.46 (b) The graph is concave up for x < 2 and concave down for x > 2, and the derivative is undefined at case ifthe graph is vertical at x = 2. One possible graph is shown in Figure 2.47 4 s@) - z Figure 247 This isthe 1, (a) The graph of @ against does not have a break at t = 0, so Q appears to be continuous at t = 0, See below. @ Nn. 7 = (b) The slope d@Q/dt is zero for t < 0, and negative forall ¢ > 0. Att = 0, there appears tobe a comer. which does not disappear as you zoom in, suggesting that Fis defined forall times # except t = 0. 12, (a) Notice that B isa linear function of r for r < ro and a reciprocal for + > ro, The constant Bo is the value of B at r= ro and the maximum value of B. ” Bo | because there is no break inthe eraph there. Using the formula for B, we have (b) Bis continuous ar tin B= £209 = Bo and tn, B= By = By, (0) The ction sot iene atr and the slope is negative forr > ro. "9 because the graph has a commer there. The slope is postive for r < ro 92 Chapter Two ‘SOLUTIONS 13, (a) Since lim B= kro and lim, B= amd \ve see that Bis continuous at ro. (b) The function E is not differentiable at r and the slope is negative for r > ro, © “ a an) (a) The graph of g(r) does not have a break or jump alr Fact that 2, and So g(r) is continuous there. This is confirmed by the 9(2) = 1 + cos(r2/2) = 14(-1) 20 40 the value of g(r) as you approach r = 2 from the left isthe same as the value when you approach r = 2 from the right. (b) The graph of g(r) does not have a corner at r = 2, oven after zooming in, so g(r) appears to be differentiable at 0. This is confirmed by the fet that cos(r/2) isa the bottom of 2 ough at r = 2, and so its slope is 0 there ‘Thus the slope tothe let of r= 2 isthe same as the slope tothe right of r 15, (a) The graph of ¢ does not have a break at y 0, and s0 6 appears to be continuous there. See figure below. * =I r 2 (©) The graph of @ has a comer at y dlilferentible at y = 0. 16, We will show f(x) Which dees not disappear as you voom in, Therefore @ appears not be is continuous atx = e. Since f{e) = 6, we need to show that that is, since (2) = 2, we need to show Pick any € > 0, then take § = ¢. Thus, ley = el 0. 18, If cis im the interval, we know lim f(z) = f(e) and lime-e 2) = g(e). Then, ‘mn (2) + 9(2)) = lim (2) + lim 2) by init propeny 2 = fc) + 9{c). so f +g is continuous atz = ¢. Also, ‘ae (f(zoCe)) = Hin f(2) im gr) by timit propery 3 = flelgle) so fy is cominuous atx = c. Finally, Haste L02) by init propeny 4 imene gz) = LO aL ixcominous tz = ae 9 Solutions for Chapter 2 Review. Exercises I Le) 94 Chapter Two ‘SOLUTIONS 4 z sey Ss ra) —+ 6. i | re) 7. Using the definition of the derivative ‘4 Sleth)~ fe) f= in (EHAE te th ~ (Or? +2) rey n ig DEH RMAY e b~ Set tng OZR SH? = Jim (U2 + 5h-+ 1) = 10+ 1 A. Using te definate date we ave e=h) = la) ge star llGat)- G+] tera = ing (Ga-2) rete) in, Tee) iumerm = = Bun nia) 9, From Table 2.17, it appears the lm 0.015, -0.01 0 for 1 < x < 3.3 and thatthe graph of f(2) is coneave up. 23, Using the approximation Ay = f'(z)-Az with Ax = 2, we have Ay = f'(20)- 6.2.50 $(22) = $(20) + f'(20)-2 = 345 +6-2 = 357. 24 (a) 98 Chapter Two ‘SOLUTIONS (8) The slope off appears tobe somewhere between student A's answer and student B's, so student C's answer, halfway in between. is probably the most accurate. (6) Student As estimate is f'(2) = AE*H=L2 white student B's estimate is f"(e) = HD={l2=9, Seudent C's estimate is the average ofthese to. oF Cs L[f(e+h)- fle) | fle) rove} [testo denen eon} _ He+h)- fle) Th This estimate isthe slope of the chord connecting (x — h, f(2 — h)) to (x + ln f(x + h)). Thus, we estimate that the tangent to a curve is neatly parallel toa chord connccting points P units 10 the Fight and let as shown below. 25. (a) Since the point A = (7,3) ison the graph of f, we h (b) The slope ofthe cage ine touching the curve atx Sto = Fin Thus. £0) 26, Al point A, we are told that 2 = 1 and f(1) know 2) = 1~ 0.1 = 0.9 andz =1+0.1 ‘Now consider Figure 2.57. Since f"(1) = 4 3.Sine 1 ihe slope of the tangent line AD is. Since AB = 0.1, Since h = 0.1, we wehave 2 = Land ya = Rie _ 2D Roo OT 2. Therefore ys = 3-02 = 28 and ys =3-+0.2=82 s0 BD = 2(0:) = SOLUTIONS to Review Problems for Chapter Two 99 27. possible graph of y = f(x) is shown in Figure 2.58. Figure 258 28, (a) The yam is cooling off so Tis decreasing and f"(t) is negative. (by Since f(t) is measured in degrees Fahrenheit and ¢ is measured in minutes, df/at must be measured in units of °F/min, 29, f(10) = 240,000 means tht if the commodity costs $10, then 240,000 units of twill be sold. f'(10) = ~29.000 means that if the commodity costs $10 now, each SI increase in price will eause a decline in sales of 29.000 units. 30, The rate of change ofthe US population is P"(t). so P'() = 0.8% Curren population = 0.0080) 5. 7'(0.5) = £08) = S04) ; ‘ro 1005) 0 is ee (0.6) ~ f"(0:3) H15 (b) Using the valves off” fom part (a. we get £"(0.6) « LE TA a (€) The maximum value of fis probably near x = 0.8, The minimum Value of fis probably near 1S the following values of 6: 3 I (ad f'(0.6) = 32 By tracing ona caleuator or solving equations, we Fore = 0.1.8 < 0.1 Fore = 0.03.5 < 0.05. For < = 0.0007, 8 < 0.00007. 133, By tracing ona ealewlator or solving equations, we find the following values of 5 Fore = 0.1.8 <0. Fore = 0.001.8 < 0.0447. Fore = 0.00001, 6 < 0.00447. BM. (a) Slope of tangent line = linn. MEE. Using h = 0.001, tangent Tine is about 0.25. 0.249084. Hence the slope of the o = = m(r—n) y—2=025(r - 4) y-2= 0230-1 y (o) fe) =k" If (4,2) ison the graph of f, then f(4) = 2,s0k- 4 = 2. (4) To find where the graph off exosses then line y = 0.252 +1, we solve: 05 0.25241 s7 = 248 oo-8 = 0 (e-Hle +2) ‘Therefore, (2.01.5) is he other point of intersection. (OF course, (4,2) isa point of intersection; we know that from. the star.) 100 Chapter Two ‘SOLUTIONS 8) The slope of the tangent line at (0, TO) is zero: itis horizontal The slope ofthe angen line at (V/19.0) is undefined: it is vertical. (b) The slope appears to be about 4, (Note that when x is 2, y is about —4, but when + is 4, y is approximately ~3.) (6) Using symmetry we ean determine: Slope at (~2, VTB): about §. Slope at (~2,—V5}: about (2. VIB): about — 36, (a) LV by IL, (2), (2) Le) IN, (DL 37. (@) The population varies period ly with a period of 12 months (ie. one year) 5000: 4000: 3 6 9 12 15 18 21 28 April July Oct Jan April July Oct Jan April (b) The herd is largest about June 1" when there are about 4500 deer. (6) The herd is smallest about February 1" when there are about 3500 der. (a) The herd grows the fastest about April * The hen shrinks the fastest about July 1S and again about December 15. (e) It grows the fastest about April I** when the rate of growth is about 400 deer/month, i.e about 13 new fawns per day [38 (a) The graph looks straight because the graph shows only a small part of the curve magnified greatly. (b) The month is March: We see that about the 21°" ofthe month there are twelve hours of daylight and hence twelve hours of night. This phenomenon (the length of the day equaling the length ofthe night) oceurs a the equinox, midway between winter and summer, Since the length of the days is increasing, andl Madrid isin the northern hemisphere, we are looking at March, not September. () The slope of the curve is found from the graph to be about 0.04 (ihe rise is about 0.8 hours in 20 days or 0.04 hours/day). This means thatthe amount of daylight s increasing by about 0.04 hours (about 24 minutes) per ealendar day, or that each day is 24 minutes longer than its predecessor. 39. (a) A possible graph is shown in Figure 2.59. At firs. the yam heats up very quickly since the difference in emperaure between it and its surroundings is so large. As time goes by. the yam gets horrer and hoter, is rate of temperature increase slows down, and its temperature approaches the temperature of the oven aan asymptote. The graph is thus concave down, (We are considering the average temperature ofthe yam, since the temperature in ts center and on its surface wil vary indifferent ways.) ‘SOLUTIONS toRevew Problems for Chapter Two 101, 2c Figure 2.59 If the rate of temperature increase were to remain 2"/min, in ten minutes the yams temperature would increase 20° from 120° to 140°. Since we know the graph is no inca, but concave dows, the actual temperature is between 120° and 140° (€) In 30 minutes, we know the yam increases in temperature by 45° at an average rate of 45/30 = 1.5°/min, Since the ‘graph is concave down, the temperature at? = 40 is therefore between 120 + 1.5(10) = 195° and 140°, (@) If the temperature increases at 2°/minute it reaches 150° after 15 minutes, at = 45, Ith temperature inereases at 1.3°/minute, it reaches 150° after 20 minutes, at ¢ = 50, $0 ¢ is between 45 and 50 mins 40, (a) We construct the difference quotient using erf(0) and each ofthe other given values: rf) = erf(0) _ ames 84270079 cerf(0.1) — erf(0) - a 1246202 er0(0.01) ~ exf(0) ee 128542. Based on these estimates, the best estimate is exf (0) = 1.12; the subsequent digits have not yet stabilized. (©) Using ef(0.001), we have sf(0.001) ~ enf(0) ooo = 11888 anu othe best estimates now 1.1283. 4L @) Table 221 2 20 /'(0) 5 | eshte) 7 7.00000 | 10000 03 Tot6a5 | Loss 07 12ssor__—_ [ssa [13ssi7 1 1si67 isan 151514 [S08 (b) It seems that they are approximately the same, ie the derivative of sinh(z) = cosh(z) for x = 00.3, 0.7, and CCAS Challenge Problems 42, The CAS says the derivative is zero, This cam be expluined by the fact that f(x) = sin? x + cos? r = 1, so f'(x) isthe derivative ofthe constant function 1. The desvatve ofa eonstant Function i 20. 43. (a) The CAS gives f(z) = 2e0s? x ~ 2sin? x. Form of answers may vary (b) Using the double ans formulas for sine and cosine, we have = sin(22) S'(2) = 2eos? 2 — 2sin? S(a) = 2sin 2008 (cos? x — sin? x) = 2eos(22), ‘Thus we get 4 vin(2e) = 20s A sin(2e) = 2eos(2z) 102 ChapterTwo /SOLUTIONS 44, (a) The frst derivative is (2) Form of answers may vary (b) Both graphs get narrow as a gets larger; the graph of gis below the z-axis along the interval where g is concave down, and is above the 2-axis where gis concave up. See Figure 2.60. o"(z) rfote) et =? Figure 2.60 (c) The second derivative of function is postive when the graph ofthe function is concave up and negative when it is concave down. 45, (a) The CAS gives the same derivative, 1/2 in all three eases (b) From the properties of logarithms. g() = In(2x) ‘J(z) + In2, So the graph of g isthe same shape as the graph of f. only shifted up by ln 2. So the graphs have the same slope everywhere, and therefore the (wo functions have the same derivative. By the same ressoning, he) = f(2) + In 3. so h and f have the same derivative aswell. 46. (a) The computer algebra system gives ie(2? +1) 4 eta) = Gre? +0) Eee +p) = ore? +1) fee gute? (by The pattern suggests that, Later emae 0! ‘Taking the derivative of (x? + 1)" with a CAS contirms this. Baines cone a Heosr = sine a tp =sin’ 2 sin reos.) = coné a ~ sin? = Deas? x = {(b) The product ofthe derivatives of sin z and eos x is cos 2(— sin #) =~ cos sin x. On the other hand, the derivative ofthe product is cos? x ~ sin? 2, which is not the same. So no the derivative ofa product is not always equal 1 the product ofthe derivatives. (CHECK YOUR UNDERSTANDING 108, CHECK YOUR UNDERSTANDING 1‘ 4 False, For example, the car could slow down or even stop atone minute after 2pm, and then specd back up to 60 mph at ‘one minute before 3 pm, In this cxse the car Would travel only few miles during the hour, much les than 50 miles. False, ls average velocity or the vime between 2 pm and 4pm is 40 mph, but the car could change is speed a lo during that time period, For example, the car might be motionless for an hour then go 80 mph forthe second hout. In that case the velocity at ? pm would be O:mph, ‘True, During a short enough time interval the car can not change its velocity very much, and soit velocity will be nearly constant, It wil be nearly equal tothe average velocity overthe interval ‘Truc. The instantancous velocity isa limit of the average velocities, The limit oF a ennstant equals that constant ‘True. By definition, Average velocity stance traveled Time, 6, False, Instantaneous velocity equals limit of difference quotients. 7. False. All we know is that if his close enough to 22 16. mm 18, 1. 20, 2. 3. True. This is scen graphically. The derivative f(a) is the slope of the line tangent to the hen {(/) will be as clase as we please to L. We do not know how ‘lose would be close enough to zero for f(h) to be closer to L than is (0.01), It might be tha we have to get alot closer than 0.0001. Iris even possible that f(0.01) = Z.hut f(0,0001) # L so f(h) could never get closer to L than f(0.01) raph of ft the point P where = a, The difference quotion (f(b) ~ f(a)}/(b — 0) isthe slope of the secant line with endpoints on the graph of f ‘atthe points where x = @ and x = D, The tangent and secant lines erossat the point P. The secant line goes above the tangent line for > « because fis concave up, and so the secant line has higher slope, ‘True. The derivative of a function is the limit of difference quotients, A few difference quotients can be computed from the table bu the limit ean not be computed from the table False. If (2) isinereasing then f(x) is concave up. However, f(r) may be either increasing or decreasing. For example, the exponential decay funetion f(2) = e~* is decreasing but f'(r) is inereasing because the graph of fis eoneave up. False, A counterexample is given by f(z) = 5 and g(2) #2) = 92) =0. 10, wo diferent functions with the same derivatives: . True. The graph of a Fincar function f(z) = mur + b isa straight line with the same slope m at every point. Thus £@) forall. 3. “True. Shifting a graph vertically does not change the shape ofthe graph and so it does nt change the slopes of the tangent lines tothe graph oirr 66°30". We estimate the slope immediately 0 the left of zp. We want to calculate the following: tiny S044) = S20) a i We approximate i by aking ro = 66.5 and h = 0.1. - 0.01. ~ 0.001 5(66.19) ~ $(66.5) _ 22.3633~ 24 _ 1, 9g [== See (65.499) ~ $(66.5) _ 234826-24 _ 5 gg 0.01 “S001 §(66.4999) - (66.5) _ 2.8VI0~ 24 _ 465 9 —0.001 *=0.00T These approximations suggest that, for ry = 66.5, finn 52044) = S20) lin 5 does not exist, This evidence suggests that S(x) is not differentiable at zo. A proof requires the techniques found in Chapter 2. (a) (i) Estimating derivatives using difference quotients (but other answers are possible) (1999) = 21910) = PCO) _ 92.0 ~ 76.0 _ 1.6 million people per year a P'(1945) = Pussy Pee) 10 9 million people per year (Gi) The population growth was maximal somewhere between 1950 and 1960. Giiy Pr950) = use F880) 3 million people per year, so P(1956) = P(1950)+ P'(1950)(1956 ~ 1990) = 150.7 + 283(6) © 167.7 million people Civ) Ifthe growth rate between 1990 and 2000 was the sume asthe growth rate from 1980 to 1990, then the total population should be about 271 milion people in 2000. 108 Chapter Two ‘SOLUTIONS (b) (i £-*(100) is the point in time when the population of the US was 100 million people (somewhere between 1910 and 1920). Gi) The derivative of f-1(P) at P = 100 represents the ratio of change in time to change in population. and its units are years per million people. In other words, this derivative represents about how long it took for the population to increase by | million, when the population was 100 million, Since the population increased by 105.7 ~ 92.0 = 13.7 million people in 10 years, the average rate of increase is 1.37 million people per year. If the rate is fairly constant in that period, the amount of time it would take for an increase of 8 million people (100 million — 92.0 million) would be 8 million poople ap hmlion people _~ 5.8 years ~ 6 years T.37 million peoplelyear ~ °"* Yours * O¥ Adding this to our starting point of 1910, we estimate that the population of the US reached 100 rnillion around 1916, ie. f-#(100) = 1916. (iv) Since it took 10 years between 1910 and 1920 forthe population to increase by 105.7 — 92.0 = 13.7, million people. the derivative of ~!(P) at P = 100s approximately 10 years T5Trnion popte = 0-78 Yearsnilion people (©) G) Clearly the population of the US at any instant i an integer that varies up and down every few seconds as a child is born, a person dies, or a new immigrant arrives, So f(t) has “jumps.” its not a smooth function. But these jumps are small relative 10 the values of {so f appears smooth unless we zoom in very closely on its graph (to within a few seconds). Major land acquisitions such as the Louisiana Purchase caused larger jumps in the population, but since the census is taken only every ten years and the territories acquired were rather sparsely populated, we cannot see these jumps in the census data, Gi) We can regard rate of change of the population for a particular time ¢ as representing an estimate of how much the population will increase during the year after time t (ii) Many economic indicators are treated as smooth, such as the Gross National Product, the Dow Jones, Industrial Average. volumes of trading. and the price of commodities like gold. But these figures only change in increments, not continuously. AxsoLuTIONS 107 CHAPTER THREE Solutions for Section 3.1 Exercises 1. The derivative, f'(«).is defined as If f(2) = Tuten ‘The definition ofthe derivative says that Hx) = tim LEtM = fl) Fe) jn, EL Therefore. cx) = tiny UTE A) +11) — 72 +11] F (2) = fim, Fr 17. Dividing gives ot) 18, The power rule gives f'(x) = 2029 — 7 1, 31 19. (we) = Gu"! 4 Sr? (w) = Gu" + 5 108 Chapter Tree /SOLUTIONS 2. me jy 29 Since 4/3, + and bare all constants, we have wea 8 Se = Jaton = Gard 30, Since w isa constant times q, we have dw/dg = Sat? 31, Since a 6 and ¢are all constans, we have BE = a(20) +1) +0 = 200+ b, 32, Since a and b are constants, we have Problems 36, So fr, we cam only take the derivative of powers of andthe sums of constant mokiples of powers ofz. Since we cannot write V3 inthis form, we eannot yet tke is derivative, 237. The isin the exponent and we have't eared how to handle that yet. 8, gfe) = 22°94 a4”, by the power and sum rules By 40, We cannot write sf; asthe sum of powers of » multiplied by constants 2. (power rule and sum rule) 41, yf = =2/829. (power rule and sum rule) Bf (Q=6F 843 and a P(e) = 12x + 122-2321 12n? + 122-2420 122 +2-2)>0 12(x + 22-1) 20. Hencer>1 or r<~2 44, Decreasing means f'(2) < 0: (2) = 42" = 128? = 4r*(r -3), 50 f'(2) <0 when 2 < Sand x #0, Concave up means f"(1) > 0: $"Ge) #122? — 240 = 12(2 = 2) SA soLuTONs — 108 s0 f"(2) > O when 1222-2) >0 #<0 of r>2 0, both conditions hold fore < Oor2 0: oY — 5g B(e'=1) >0 so e8>1 so r>lorz<-i “The graph is concave up when d?y/dz? > 0: #y & 27> 0 so r>0, ‘We need values of x where {2 > 1 or x < ~1} AND {2 > 0}, which implies x > 1. Thus, both conditions hold forall values of larger than I. 46, Since f(z) = 1° ~ 62" ~ 152-420, we have f'(2) = 32° — 122 — 15. To find the points at which f"(r) = 0, we solse Br? 12-15 =0 32” —4r—5) =0 32 +e 5) =0. We see that (2) = O at 2 = —1 and atx = 5. The graph of f(z) in Figure 3.1 appears to be horizontal at x = —1 and at z = 5, confirming what we found analytically. Se) Figue 3.4 7. F(a) = 84 2vir S'(r) = -8+2v9r =4 ra? aay 48. (a) Since the power of z will go down by one everytime you take a derivative (until the exponent is zero after which the derivative willbe 270), we can see ely tha f(z) = 0. (by fe) = 7-6-9-4-3-2-1- 2° = 5080 49, Differentiating gives f(z) = 62-42 so f'(1)=6-4=2, ‘Thus the equation ofthe tangent ine is (y — 3) = 2( ~ 1) or y = 2x ~ 1. 50, (a) We have f(2) = 8, so a point on the tangent line i (2.8). Since "(x m= #2) = 30)" 32, the lope ofthe tangent is given by 2. Thus, the equation is 8= 122-2) or y= 12-16. 110 Chapter Tee SOLUTIONS (by See Figure 3.2, The tangent line lies below the furetion f(2) = underestimates. 9, so estimates made using the tangent line are y= 12216 Figure 32 Si, The slopes ofthe tangent fines to y = 2? ~ 2 + 4 are given by y 2x ~ 2. A line through the origin has equation mat. So, atthe tangent point, 2? — 2x +4 = ma where m n= 2 Bn 2et4 = (Or 2)r e242? Oe -r+4=0 (e+ Mr-2)=0 r=2.-2, ~Gr, respectively. Graphically. this can be seen in Figure 3.3 ® (2,12) yaa Qa) Figue 3.3 52, UF f(2) = 2". then J (2) = nz". This means f'(1) =n 1"-1 = n- 1 = n, because any power of I equals 1. 52, Since f(z) = a2”, f'(z) = anz*“!, We know that (2 S.and f'(4) = (an)s"-? = 24. Therefore, £0) FO) (anja! fay? ne ~ (3) 2°" = 8, and thus » Substituting n = int the expression for f'(2), we get = a(4)(8), ora = 9/32 31S0LUTONS 111 $4. Ye. To see wy wesubinte y= 2 noth equation r= We casi ME = a8) = nat Te diferena eqution becomes 13a(nr*~') = x” Bur 18o(ne"™!) = 19n(2- 29-1) = 1S” s0 we hve Ian(e") = 2" ‘This equality must hold forall x. so we get 131 = 1, s0 1 = 1/13. Thus, y = 2!" isa solution ‘SS. Since f(t) = 700 — 3¢”, we have (5) = 700 — 3(25) = 625 em. Since f(t) = —6t. we have f'(5) = —30 env/year. Inthe year 2060, the sand dune was 625 cm high and was eroding a rate F 0 centimeters er yea. 56. (a) Velocity u(t) = #8 = £(1250 — 16°) = ~32r Since ¢> 0 the bls veloc i negative. This is retonble, since is eighty is decreasing. (b) acceleration a(t) = 4 = 4 (228) = ~92. Sots acceleration ie negtive constant ~92. (6) Theball his the ground when is height y = 0. This gies 1250 — 16" = 0 f= 48.84 seconds We discard ¢ = ~8.84 becouse ime # fs nomtegate, So the al its he ground 8.84 seconds fier is release, at ‘which ime is velocity is (8.81) = ~32(8.84) = ~282.88 feevsee = —109.84 mph. 57. (a) The average velocity between t = 0 and 1 = 2is given by £(2) ~ f(0) _ -49(2") +252) 43-3 _ 334-3. average velocity = LQ} £0) _ SAS) eet =e 15.2 msec, () Since f'(0) 8 + 25, we have Instantaneous velocity = #2 =9.8(2) +25 = 5.4 mbec ) Acceleration is given f(t) = ~9.8. The aeceleration at which is -9.8 msec’ (a) We can usea graph of height against time to estimate the maximum height of the tomato, See Figure 3.4. Alternate wwe can find te answer analytically. The maximum height occurs when the velocity i zero and v(t) = ~9.8¢+25 = 0 When f = 2.6 ses, Atthis time the tomato is ata height of f(2.6) = 34.9. The maximum height is 34.9 meters, 2 (and al other times) s the acceleration due to gravity, 26 52 Figure 3.4 (€) We see in Figure 3.4 that the tomato hits ground at about 1 = 5.2 seconds. Alternately, we ean find the answer analytically. The tomato hits the ground when I(t) = ~A90 + 200-43 112 Chapter Tree /SOLUTIONS We solve for fusing the quadratic formuls 254 (OFT A=49) 25 JOBE, =< t= 012 and 1252, ' ' ‘We use the positive values, so the tomato hits the ground a | = 3.2 seconds, Fo (Maho 2 (rt) aT (1 Since SF is positive, the period T increases ay the length # increases 60. ta) by This is the formula for the circumference ofa circle. (0) A'(r) = AEEM=AE? for small h. When ft > 0, the numerator of the difference quotient denotes the area ofthe region contained between the inner circle (radius r) and the outer circle (radius r + h). See igure below. As h approaches 0, this area can be approximated by the product of the circumference of the inner circle and the “wih” ‘of the region, ic. . Dividing this by the denominatr,h we get ’ = the circumference ofthe circle with radius 7, & We ean as think about he derivative of asthe rate of change of area Fr asl change in radius. Ihe radius increases by any amount the aca wll neease bya thin ring whose area is siply the crcunference a that radius times the small amount, To get the rate of change. we divide bythe sll amount and aban the circumference 1°. itleremiatng gives $2 = 4r® = surface sea ofa sphere. The difference quotient “#42= isthe volume between two spheres divided bythe change in us, Funher more. when is very smal the diffrence between volumes. (r+ h) ~ V(r). s lke a coating of paint of depth applied tothe surface ofthe sphere The volume of the pein is about fh «(Surface Arca fr small: dividing by h gives back the surface area “Thinking abou the derivative asthe rate of change ofthe Function for small change inthe variable gives another ‘vay of sccing the resul Ifyou increase the radius ofa sphere a small amount, the volume increases by avery thin layer ‘whose volume isthe surface are that radius ipled by that small amount 62. oY ‘simtlet Fh = Ei [zeby] we a2SoLUTONS 113 Me) 0 fig, tN = 2 de Ih a jen = hae [ear 1 RS = toh [Beene tin L [aCe She? + ante + 0) = bh Fee hP = ting | [rBhe? = Beh? =H? oh | e+ Ay - et letra avoh [fe +hy a eee] Fe +h ‘ers ivolving 4 and higher powess of AGF = RT Solutions for Section 3,2 Exercises 1. f'(e) = 2" +22, t+ de! 1n5)5* (4n2)2" + 2(1n3)3" dy | Fast & = sw ojo" - a 4y 5 away & Gp = 3-Alndye, Lange - 382 9, Y= Lana? - Bord) 114 Chapter Tee (SOLUTIONS » F(a) (Inder. (nays, 16. f'(t) = (In(la3))(tn3y 1 #2) = 3° +3" ns ty 5g 18, =. 5'ind+6-6'In6 vw, Beeting 20, h'(2) = (In(lu2))(lu2)*. 2b. f(z) = (Ins)s" 22. This isthe sum of an exponential funetion and a power function, so f(z) = In(x)a” + x7" 2. (2) = 0" lat ar A f(z) = ZOO 4 (n?)F Ine?) 28, f'(2) = (21n3)z + (In d)e* 2 sie)= Lore 4a —ona4 Se sath 2+ (In2)2* gevh im b(4yt = gy + inh)" 29, We can ake the derivative of the sum 2? + 2°, but not the product. 30. Once again this isa product of two fonctions, 2* and 2, each of which we can take the derivative of; but we don’t know how 1 take the derivative ofthe product. BI. Since y= ee? yf = he? =e. 32 ya ol = (€*)*. 504 = Info?) (68) 233, The exponent is 2”, and we haven't eared what todo about that yet 3 FC) = (ln VIVA = (ln 2)2* 3S, We can’t use our rules if the exponent is VB. Probiems 36. we SF = 25,000 -(in0.98)(0.98'). ALt = 23, this fs 39,000(In 0.98)(0.98") = —<44.328, (Note: the nevative sign Indiate thatthe population is ecreasng.) 37, Since P= 1+ (1.05), 42 = In(1.05)1.05'. When ¢ = 10, ap SF = (tu 1.05)(.05)"° = $0.07947/year = 7.95¢/vear. 3K, Weave f() = 5.3(1.018) so fe) = 8.(10 1.018)(1.018) = 0.095.018). Therefore (0) = 5.3 billion people and (0) = 0.095 bition people per year: a2sounioNs 118 In 1990, the popelation of the world was 8.3 billion people and was increasing ata rate of 0.095 billion people per year We also have (30) .3(1.018) = 9.1 billion people, and £' (80) = 0.095(1.018)*" = 0.16 billion people per year. In the year 2020, this model predicts tha the population of the world will be 9.1 billion people and will be increasing at ‘arate of 0.16 billion people per year (1.95) In 1.39 = 22.5(1.35)" 3.050. Thus the value ofthe car ater 4 years is $13,080. (b) We have afunetion of the form f(t) = Ca’. We know that such functions have a derivative of the form (Cn a) a" ‘Thus, 1(¢) = 25(0.85)* In 0.85 = ~4.063(0.85)*. The units would be the change in value (in thousands of dollars) ‘with respect to time (in years), or thousands of dollars/ear. (©) ¥"(A) = —4.063(0.85)* = —4.063(0.522) = ~2.121. This means that at che end of the fourth year, the value of the car is decreasing by $2121 per year. (2) V'(#) is positive decreasing function, so thatthe value of the automobile is positive and decreasing. 1"(t) is a ‘negative function whose magnitude is decreasing, meaning the value of the automobile is always dropping, but the yearly loss of value isles as time goes on. The graphs of V(t) and V(t) confirm thatthe value ofthe car decreases ‘with time, What Uhey do not take into account are the casts associated with owning the vehicle. Atsome time. f, is likely thatthe costs of owning the vehicle will outw ‘may no longer be worthwhile to eep the car. AL, (a) The rate of change ofthe population is P(t) It P(t) proportional to P(t), we have +P). (b) If P(t) = Ae then P'(O) = keel 42, (a) f(z) = 1 =e crowes the ans where 0 = 1 ~ e%, which happens when e* = 1,50 4 = 0, Since f"(2) = $0) =i () y=-2 () The negative ofthe reciprocal of ~ 4B. Since y = 2, y/ = (In2)2*. At (0.1). the tangent line has slope In 2 so its equation is y = (In 2) + 1 Ate y De Lose = ~ py |. 50 the equation of the normal line is 0,80 art torte fx) fiz) fz) /"(Q), etc. we have 2a = 1,6= Land So, using g"(0) figure below. cand thus o(2) = $2? + 24+ 1, as shown in the jearer ‘The two fmetions de look very much alike near 2 “They both increase for large values of, but e* increases ‘much more quickly, Fr vey negative values of 2, the quadratic goes 10 20 whereas the exponential goes to 0. By choosing, 8 funetion whose first few derivatives ugreed with the exponential when x = 0, we gota funetion which looks lke the ‘exponential for -values near 0. 118 Chapter Tree SOLUTIONS 45, The derivative of ef is (et) = ef. Thus the tangent line st x = 0, has slope ® = 1, and the tangent line is y = 2 + 1. A Tunetion which is always concave up wil always slay above any ofits tangent ines. Thus €® > 2 + 1 forall z, as shown inthe figure below. 446, The equation 2 = 2 has solutions x = 1 and x = 2. (Check this by subsuting these values into the equation). The ‘raph below suggests that hese are the ony slutons, but how can we be sure? Lars lok atthe slope ofthe curve f(z) = 2, which is f"(z) = (n2)2¥ = (0.698)2, and the slope ofthe ine ‘o(e) = Bx which is 2. AUzZ = 1, the slope of /(2) is ess than 2 the slope of f(2) is more than 2. Sines the Slope of f(2) is lays increasing there can be no other point of itersetion, (If here were another point of itersetion, the graph f would have to “umn around Here's another way of seing this, Suppose g(2) represents the position ofa car going a steady 2 mph, while f(=) represents &car which tarts abea of g (because the graph of fis above g) ad is iiialy going slower han g. The eat J ists overaken by g Al the while, however, is speeding up uni eventually it ovenakes 9 again. Notice thatthe wo cas wll only meet twice corresponding tothe two intersections ofthe curve: once when g overakes f and once when fF overakes g 47, For x = 0, wehave y = Differentiating gives Land y=140= | s0 both curves go through the point (0,1) forall values ofa. Mat) =a a Ina =In0 = a inal aa+s) dr ‘The graphs are tangent at x = Oif Ina Solutions for Section 3,3 —_—_—_—_—_—_—_—_—— Exercises 1. By the product rule, f(z) = 2e(z% + 5) +2°(B2") = 2x" + 3x4 + 10r = rt + 10x. Altematively f(z) = (e!+ br!) = 5° 4 10x. The wo answers should, and d, match. Using the product rule, F(a) = (n2PH’ + (1u3)2HF = (In2-+ In)" 3" 2-3)" = 6", 1n(2-8)(2-3)* = (In6)6* or, since 2° 3° P= YF “The wo answers should, and do, match. Bfl)aret tet lae(etl) 4. yf = 2 + x(ln2)2 = 27(1 + In2). aasoumons 117 8. y= shyt + vena. 6 (2) =(@=2)-9°(103) 49" (22 Let) ast [ewave? == (: 8 aad (0 a3! =e? 41149. 9, yf = (BE = Lae! + (PTF bye! = (4 ab De Laree? _ (1-2) er e = Sr = 252" Sr eye) 8 M0 = Ty > TF Gera > Gear (b+ aN(t+3) (C4542) _ Pe Or+13 ae Cyr F dz _ +341) +242 17. Using the quotient ue gives $F = FOES DAES A ED oy a 18, Divide and then differentiate 847 w= 2y-6.y x0. yal +)- vie Crs £(%%)- 2. \ a 2 Vie Vie afr (Qr)Qr +1) = 2? _ 2r(r +1) ae (oat) ~~ Grate 7 Orsi 24, (Notice that you ean cancel a = out ofthe numerator nd denominator to get a Rey f@)= 2#0 Then [If you used the quotient rule correctly without canceling the = out fist, your answer should simplify to tis one, but itis usually a good idea to simplify as much as possible before diferentating.) 118 Chapter Tree SOLUTIONS 25, we) = MAD Watery” _ Mere = 3 Be 2p) = 2226") = dQ + 7°) 26 EO) = Soap a. pte) = 24S +4) — 42) 4 88) O+ar+ary 24 Sr tas” ~ 3 11z ~ 82" 7 C++ tz? ~ 82-1 Bes aay 228, We use the quotient rule, We have seq = (ett RNa) —(an-+ 00) _ ace f= ae 20, wl = (BH? + 5)(t — 742) + (P+ 501287). Problems 30. SF (2) = 32x —5) +2182 +8) = Pe +1 siz) = 12 ‘31. Using the product rule, we have Since e“* > 0, forall z, we have f"(z) < Of r —2 <0, thatis.2 <2, 32. Using the quotient rule, we have vg) = 9x12) =e Gap ~ +P (e+ 1) + Delta? +42) wai n2e" 41)? +8r"(e" +1) area (= +1) +82 Fr 227-1) a Since (2° + 1)? > 0 for all x, we have g"(x) < 0 if (327 — 1) <0. or when -Ase< 3. Since f(0) = ~3/1 = 5, the tangent line pases though the pint (0. —5). its vical intercept is —B. To id the Shops ofthe ange tne. we tnd the dersatve of f(z) wing the quien ale fey GaP wy ‘Att = 0, the slope of the tangent line is m = (0) = 7. The equation ofthe tangent line fs y = 7 ~ 3, assownons 118 3% Sx) = ee ey + (Ye dete + e%e* (rom Problem 35) = 3%, 37. Since fhe" = 26% and Ze™ = Se, we might guess hat Ze = se! 38. (a) Although the answer you would get by using the quotient rue i equivalent, the answer looks simpler in this case i {Yu just use the product rule: aa?) dz) _ da “= ae ae) d(x) | d(z) az) | d(x") Var de “de de dr) Az) , dl He +s [et 2 ae) 4, jadle) , ade) ids peteds eacatecy Since gap we diferente vo obtain ‘ Bey 2”. Now solve for d(x'/*) 420 Chapter Tee (SOLUTIONS AL. a) We have h4(2) = f'(2) + 9 {Weave W() = Jal) + 429 (2) = 3) 496-2) = ey Leu) = FAW) SH) _ 8 fe) We have W/(2) oon = B= 42. (ay (2) = F(S)H(=) + HDF (2).50 GO) = FOG) + HIF) = 4-149-5=18 (IHG) — HWAF CW) 5g gy) — $0) = 365) __ ) Cw) Tia 063) n $B. (2) = 102! "eis ofthe form gh + h'g, where gle) = 2°, ol(2) = 102? and ha) =e" Wa) =e" “Therefore, using the product rule let f = gh with g(x) = 2° and h(z) = e*. Thus fle : 44. (a) f(140) = 15.000 says that 15,000 skateboards are sold when the cost is S140 per board. 4'(140) = —100 means that ifthe price i increased from $140, roughly speaking, every dollar of inrease will Sezrease he tl es by 10 boards ara d 0) Fp p= gy PLOY = 10) + 2F se = (140) +140 '(140) = 15,000 + 140(—100) = 1000. (©) From) we sce that 2] = 1000 > 0, Thi means tht the evenue wil increase by about $1000 ifthe price is raised by SI 45, We want dR dry. Solving for R: +m hi rire A= BEE. which gives = OE So, thinking of ro asa constant and using the quotient rule, ak _ ralreen) a etn? we 46. 4a) Ifthe muscum sells the painting and invests the proceeds P(@) at time t, then ¢ years have elapsed since 2000, and the time span up to 2020 is 20 ~t. This is how long the proceeds P(t) are earning intrest in the bark. Each year the ‘voney isin the bank it earns 5% interest, which means the amount in the bank i multiplied by a factor of 1.05. So, at the end of (20 ~ f) years, the balance is given by Bt) = PCO(I + 0.08)°"" = P(E)(1.05) o Ble) = P(I.08)°%(4.08)~ (€) By the quotient rule, BW) = (1.05) (.08)" SSOLUTONS 121 So. vay = (05) | 3000{0-08)"° — 150,0000(1.05)"® In 1.08 B00) = (1.05) | ON ee = (1.05)"(5000 ~ 150.0001 3.05) = -3776.65. A, Note first that f(w) iin WEEE and w isin Be (a) g(v) = poy (Thisis in AB.) Differentiating gives i) = EC (= SEO I Foy so, : (80) = ghz = 208. 1160) = 72 P - (80) = Sa ~ EA for each 1 increase in sped (by h(w) = w- fe). (This iin B= HBS = Has.) Ditferen Me) = fo) +e f 0). 38 gives 90) = 80(0.05) = 4882 (80) = 0.05 + 80(0,0005) = 0.09142 for ech 13 inerease in spe art (3 tells us that at 80 ka, the eat ean go 20 km on | liter. Since the frst derivative evaluate at this velocity is negative, this implies tht as velocity increases, fel effceney decrease, athighe slates theca will not 30 8 for on | ltr of pss Part (b) els us that at 80 kv, the eae uses Tiers Ina hour, Since the Gs derivative ‘valate at this velocity is positive, this ean that at higher velocities, the car wil use more gas per hour. © 48. Assume for ox) # f(z). (2) = a(2) and g(0) = 1. Then for (ey = HeEIe = ede® _ UGC) =la)) _ gle) - ae) ea cP os But, since gf) = of (2). A(z) = 0 s0 h(x) is constant, Ths, the rato of g(2) to e* i constant Since 20) 22) ust equal I forall x. Thus g(2) = ef = f(a) forall 2,0 f and gare the same fartion. $8. @) (2) = (2-2) 42-9). (by Think off the product of vw factors, with he Bist as (2 ~ 1)(r ~ 2). The reason fr thisis that we have already differentiated (2 — 1)(4 = 2)) Sle) = fle - fe ~ 2x 9). Now f(z) = [(2 = 1N(z = 2] ( ~ 3) + le = fe — Ile - 3) Using the result of 3): f(a [x= 2)4 = DY 3) + [—e- 2-4 (© = 2) 3) +(e = Ne 3) +(x Ye 2) (€) Because we have already differentiated (2 — 1)(2 ~2)(4— 3), rewrite f as the product of two Factors the frst being (=e = 2( = 3) Se) = [le = Ie le — ay] = 4) Now f(r) = [2 = De = 2x ~ Ble =) + [le = NE = 2) — BM = 122 Chapter Tree SOLUTIONS £ (2) =[(e = Me -3) + - le = 3) +(e Yle- Mle) [le = fe = (2 =3)} 4 = (6 2)le = Ble 4) + (= le 3)(e 4) $e ~ Ie 2le—4) + (= le = H(z 9). From the solutions above, we ean observe that whan fis product, its derivative i obtained by differentiating each factor in tum (leaving the other factors alone) and adding the results ). From the answer to Problem 49, we find that nee FQ) =e le) #(E = Pe = rade (erent) Le =a) Ha = rile = rade (@ = rena) Le = rer ME =m) tect Gores) (ed et os 51, (a) We can approximate £[F (z)G(2)H(z)] using the large rectangular solids by which our original eube is inereased: Volume of whole — volume of orginal sid = change in volume F(z + h)G(e + h)H(x + h) ~ F(2)G(x) H(z) = change in volume. ‘The volume ofthis slab is F’(2)G(2)A(2)h sions which are added (the long, thin rectangular boxes and the that as ft + 0, these volumes wil shrink much faster ore the smal As in the book, we will ig small cube in the corner. Ths ean be justified hy recognizing SASOLUTIONS 123 than the volumes of the big slabs and will therefore be insignificant. (Nove that these smaller regions have an h? or ‘nin the Formulas of their volumes.) Then we can approximate the change in volume above by: Fle + h)G(e + h)H(e +h) ~ F(e)G(2)H(z) = F(2)G(2)H(2)h (op slab) +F(e)G(2)H(z)h (front slab) + F()G(z)H'(a)h (other sab). Dividing by h gives F(z + h)G(e + h)Hle +h) ~ FIG) H(2) rs rs & F'(z)G(2)H (2) + F(2)G' (2) (2) + P(2)G(e) He) Letting h +0 (FGHY = F'GH + FO'H + POH. (©) Verifying, Stee) -oe- (2) = Fy + Fey =(F'G+ FG + PCH FGH + PG'H+ FGH' as before. (€) From the answer to (b), we observe thatthe derivative ofa product is obtained by differentiating each factor in turn (Ceaving the other factors alone), and adding the results. So, in general, (fi fae fave fa)! = Sifafa ft RBs In ot fa he 82, (a) Since x = « isa double nero of a polynomial P(z), we can write P(z) = (x ~ a)?Q(e),s0 Pla) = 0, Using the product ule, we have fe Pi(z) = 2x - aQ(z) + (2 - a)°Q'(e). Substituting in x = a, we see P'(a) = D also, (b) Since P(a) = 0, we know 2 = ais a zet0 of P, so that ~ a isa factor of P and we can write ‘where Q is some polynomial. Differentiating this expression for P using the product rule, we get P(e) = Q(x) + (2-2) Qa). Since we are told that P'(a) = 0, we have P'(a) = Qa) + (a - a)Q'(a) = 0 = a isazer0 of Q, so again we can write Qz) = (ea) 2). where R is some other polynomial. Asa result, Plz) = ( ~a)Q(z) = (x - a) R(2). and so Q(a) = 0. Therefore so that 2 =u isa double zero of P. Solutions for Section 3,4 mr, Exercises 1 f(a) = 992 +1)" 2 F@)= 4 = 902 +1)". (=a 8 (22) = 00(t? + 1)9*(28) = 2une(e? +1) Charter Te BOLUTONS Log(e® + 1) (Se) = B0087( + 1). 00( VE + (sin) = ROVE. 5(w* — 2w)*(4u* — 2) We can write w(r) = (+4 +1)", s0 ort wir) = 3 IF (4r) mY? = (7)? 90 7°(8) = (In nz}a*) (In 3)30247, (2 + e*)*(82" +e"). efx? + 5°] +e [De + (In5)5*] =e fOr? + 20 + (Ind + 2)5*]. cen tt? = (2t— ct)em"". Using the product rule gives v'(!) = 2te"* e842 C4907 2(1 4.31) 3. 6(1 + BCH + in which case it isthe same as problem 26. $0 f'(2) = = (va(n2)2*) _ 1 2s Ine cma Teens : yf ef + te (2) we rn2(292)(@)= st wen = (2 dr @ +i (+1 z SASOLUTIONS 125 86s He (Fey = (1n2y(8e")2" 2 HE We = TF IF) = He MCND = Ee 38. f(z) = Ge )15) + (e-* (22) = Be ~ 22em* ». . F'(e) = (2° )0%) + Gu" + 3}(e** 2) ?(5 + 5u” +3) = we” (Su? +8). 40, 4. 2 e- (9)( = 1), 8 [ee vayy] = aye. HL FO) = 20-7202 = —8(e 45. Since a and b ure constants, we have f’(t) = ac**(b) = abe". 46. Since a and bare constants, we have f(z) = 3(az* + b)*2ax) = Gaz(az® +b)? 447, We use the product rue, We have £2) = (az\(e“™*(—W)) + (a)(e) = ~abze™™ +40, 4B. f"(2) = Gale* ~ 4) + (B2" + wet = Gre — Dar + 327eF + ner, Problems 49, Weave f(2) = Tine is (2 ~ 1)® = 1, so (2,1) is point on the tangent line, Since f(x) = 3(2 ~ 1)?, the slope ofthe tangent m= FQ) = 32-1 ‘The equation of the line is (2-2) or y= 32-5. 50. the S(2) = S06 = 2e* 1) = 6 +e £(1) = Be = 20)e* ym =m(z—71) yer +e Web — 2e1}( = 1) w= Gere y de® ~ 2e"")2 ~ (B02 ~ 267?) We — 26" )z — Be? + 2071 + Ge? emt = 4451,662 ~ 3560.81, 128 Chapter Three SOLUTIONS S1. The graph is concave down when f"(2) < 0. The graph is concave down when 42° <2. This occurs when 2? < $.or Jy <2 < de 3, F(z) = Or + 1)°2)IBe — )'] + [2x +1)" Re - 19°) 22 + 1)°(Bx = 1)°[20(8r — 1) + 21(2r +1} (22 + 1)°(8r — 1/9) 022 +1) $2) = [E+ I((Az —1)8 + (2e + 1)%(6)(B" ~1)°()}(02x +1) +(2r + 1)°(3x — 1)8(102). 53. (@) H(2) = F(G(2)) (G(4)) = FQ) =1 (Giz) FG) Gite) (GG))- 6) = FQ) (F(z) (F(A) = G8) = 4 (F(z) (F(z) F'(s) a (F(A) F(A) = (8) -7 = 8-7 = 56 $4 (a) ilferemiating 92) = Ta) = (42), we have ia) = Uyteyy-"”? fe) = 92) = 5)" “a= fay = LO. 3 98 Sia Wa (b) Differentiating h(z) = f(/Z), we have MO=1WO- ae war) = Oe $ $8, (a) Since W(e) = f'(ale))-9'(2), we have HQ) = F(92))-9'@) = S'8)-9'2) (b) Since W'(2) = g (J(2))- #(@), we nave we (o) Weave (2) = f'(f(2))-f"(2).s0 r@) eee age FU): £2) = £6) FQ) = re. SASOWUTONS 127 p(z) = k(2z), #G)=¥@ see gz) = k(z +1). g'(z) =k (e+ 1)-1. FO) =k (0+ 1) ; ra)=k (Fz), ra)= raver (4 de a S51. Yes. To see why, simply plug 2 = Y2E45 int 4 or 45) = 2arrard = Zorn Hert toyd = Fors ay d= Zor gayhy at But since 2 = (21 +5)4, we have (by substitution) ad _ 432 (2,-2) _ Iefollons that 32°F = 32? (F2™*) = 2. 58. We see that m'(z) is nearly ofthe form f"(g(z)) g/(2) where Ho) bout g'(z) sof by a multiple of 6. Therefore, using the chain rl, lt Holz) _ $6 snd 9(2} m(z) = 59, We can find the rate the balance changes by dlferentiting B with respect to time: BY(t) = 5000€.0,08 = 4006 Calculating B’ atime t = 5, we have B’(5) = $596.73/y. ln 5 years, the account is generating $597 per year of interest. 60. The concentration ofthe drug inthe body ater 4 hours is 10) = 27e°° 0) = 15.4 opi. ‘The rate of change ofthe concentration isthe derivative F (0) = 27 (—0.14) =~ 3.786, ALE = 4, the concentration ix changing at arate of F(A) = ~8.78e-° 4 = —2.16 mpi per hour 128 Chapter Three SOLUTIONS and f(10) = 6e 9151 = 6.883. The derivative of f(t) is 61. We have £(0) o7ee001% F(t) = 6°™ 0.013 and so #"(0) = 0.078 anal f"(10) = 0.089. ‘These values tellus that in 1999 (att = 0), the population of the world was 6 billion people and the population was growing a arate of 0.078 billion people per year. In the year 2009 (at = 10), this model predicts that dhe population of, the world will be 6.893 billion people and growing at arate of 0,089 billion people per yea. 62. ta) 4Q _ A -00001211 at ar 0,0001216~290 rs f ol sono cooront 2. sonmeroe oon sore 2010 com @ aw na r= ltd + We! 30(—2)e" 60" 0) Sine ialay postive, 2 isms sr bec he empera of oa deeing a to Temogsinae oi a fat since e-® <1 for allt > 0 and ¢® = 1. This is just saying that atthe moment that the can of soda is put inthe the temperature difference between the soda and the inside ofthe refrigerator isthe greatest dropping the quickest. tiger coteepatir of te 64) Bm P(14 )'t (14 2) -Treoxpeion 2 tus ow athe amount of money inh tnk is ae 100, 100, dt changing with respect to time for fixed initial investment und interest rate r. ee fH P1( 14430)" lg: Texan “2 inden ow fst te anon of money changes wi rege (1-455) pl. Te een 22 nn ow fs the mou of moe hrs hese Witenes cso a alive Pie ing derivatives and © the chain rule gives 65, The ripple’ rca and radius are related by A(t) = =[r()P. da ar ard 0 cm we have ‘We know that dr/dt = 10 emisee, so when r da F+2+20+ 10cm? see = 400nem", SASOLUTIONS 129 66. (@) 0 2 rte ao ago as wth pe ote pe aq 67. (a) Fort <0. rn = Jo ge Fort > 0,0 = 32 = £8. () Fort > 0, O (hats, ast + 0"), Since I = 0 just to the lef of # = 0 and 0. (©) Qisnotifferentiable att 68, The time constant for Q isthe time. Ta, such hat Q = Qa/e. Thus, Ta satisfies —Qo/RE just to the right of € itis not possible to define J at because there is no tangent line at! Qe = oyerterte Canceling Qo and taking natural logs gives etorne 2} “To Te = 71 Ta = RC. Tofind I = dQ/dt, cterentiate Q: 4Q _ = Qo ,-ure, a RC Since he exponent of is unchanged, sos the tm constant. We krow hatte iil curtis =Qo eRe IT) isthe time constant for Z, we know RC Canceling ~Qo/ RC gives Le getune ‘This is the same equation as the one we solved for Ta, so T= RC. 130 Chapter Tree /SOLUTIONS (69, Recall that w = d/l, We want to find dhe acceleration, de/dt, when x = 2. Differentiating i respect tof using the chain rule and substituting for v gives expression for v with ad de ple +80 2). FE = (ar + Bye = (Qe + 9)(0" + 32-2) Substituting 2 = 2 gives (2) +3) +3-2-2 56 empec 70. (a) The population isinereasng if AP/alt > 0, thats, if AP(L~ P) > 0. Sine P > Oand k, L > 0 we must have P? > O and L ~ P > 0 for his tobe true, Thus the population is increasing it0 L. The population remains constant if dP/dt = 0,0 P = Oor P| (b) Differentiating with respect to # using the chain rule gives ep yy — Lag 2). dP Fe = PULP) = Seeuep — KP?) = (kt — 2ePRPUL- P) =F PUL 2P\L- P). 7. Let f have a zero of multiplicity m at ¢ = a so that Ha) = (2 aN"A(2), Aa) #0. Differentiating this expression gives SE) = (2 = a)"W le) + mlx = ay" AG) and both terms in the sum are zero when 2 = a so f(a 0. Taking another derivative gives $1 (2) = (ea) Ha) + 2a ~ a)!" PAN) + mom = Yl = ay"), ‘Again, each term in the stm contains a Factor of (2 ~ a) to some postive power, s0 at x = a this will evaluate 100. Differentiating repeatedly, all derivatives will have positive integer powers of (r ~ a) until the mand will therefore vanish. However, La) = mih(a) #0. Solutions for Section 3,5 —_—_—_——— Exercises 1 Table 3.4 = [ome | Difeense Qooient | — ain Oo] 10 0.0005 0.0 0.1| 0.993 | 0.10033 | ~o,ov08a 02| oar} 0.1906 | 0.19867 osossss1| 020 | 0.20582 oa] oszi0s | — -0.s8988 o.38042 0] o.sr7ss | — -oross | 0.701 osfosrsss] -v.s6s0s | —0.56161 SSOLUTONS 131 2. 1'(B) = cosé~ sind. 3 in sin 9 + c08 cos 6 = cos? 8 — sin? @ = cos 28. 2 = —4sin( 40). J'(2) = cos(32) 3 = 3.cos(32) @) a a Fe sin(2 ~ Sr) = cosl2 ~ Bz) 7-2 3) Using the chain rule gives (2) = 34 sin(ma). 491 (9) = 2sin(26) cos(26) 2 ~ = Asin(28)c0s(26) — (22)(cos2) + 2%(—sin 2) = 2r cos ~ 2" sin. cose!) nh ("7 )(-sinz) = ~sinze’ 12, f'(y) = (cos yet. Be ee" — G(sin Be", 4 the chain rule gives F'(B) = 3cos(38}e*™(2, 15 (6) = Sean 16, w(x) = owe) v. He) = (1 cos) F)= 40 ewe H-(-sina) Wierd 8 f'(2) = [-sinGina](cos2) 18. £0) = Samay 20, K’(2) = 3 /Sin(Be)(2c0s(2z)) = 3.cos(2r) Sime). f(x) =2- [sin(Sz)] + 22[eo0s(32)] -3 = 2sin(B2) + 6x cos(3r) sin(20) + 2e® cos(24). (e-7*)(—2)(sin 2) + (€72)(c082) = —2sin 2(e7*) + (e7* (cos) = cost 2Vant 25, yf = Sain’ Beos8. [eos - 2sin2} w= (—cos8}e™ #8, N() = 1 (cost) + #(sint) + Z sina +3cosa sia ¢080) cos? a + sin® a(3 cos? a(~ sina) = dsin* a.cost a ~ 3sin® a.cos? a a. 1 power and quotient rules gives desing cos (1 — cos.r) ~ (1 = sinz) sing (= cos)? f= 3 (SS 132 Chapter Tree (SOLUTIONS 1 fi=cosz [—cosz(i ~ cos) ~ (1 2\ T-sinz (= cose)? 2sin ros (cos? 2 +1) + 2sin ze08 2(s (eos? + 17 ‘3M. The quotient rule gives G'(z) or, using sin? x + cos? We) 2 esinzeone OO) Cote iF = £( y ) = saves ne cosytat ysiny ay (eure (cosy a)? (corutay? 36 WI (@) = (in2)2** cosz. 1 uf = (In 27" eos +e") BB f'(2) = 2eos(2x) sin( 3x) + Bsin(2e)cvs(S2). 3. f"(0) = 28sin8 + 6 cos8 + 2086 — 20sind ~ 2eord = 6 cos. 40. $'(2) = cosfeasz + sin z)(cos = Sinz) AL f{(w) = ~2e0s wsin w ~ sin(w?)(2u) = ~2(cos wsin w+ wrsin(w*)) Problems. 42, The patie in the table below allows us to generalize and say thatthe (4n)** derivative of cos is cos, je. 7m a Bee wih darvaine [sine [—eos [sine [cose] [eos | sina | ~ cone 43, We see that g(2) is ofthe form le) f'(2) = fle)-9'(2) {g(2) wih) =e dg) = xin. Tere sig he tent eet aye Mt ale) ~ Sine 4H4 Since F(z) i of the form sin, we can make anil guess tha Fle os(z), hen F(z) = ~Asin(az) so were off by a factor of =. To Bx this problem, we modify our guess by actor of —4, so the net ty F(z) = ~(1/4)cos(t). whet has F(z) =sin(tr), assowTions 133 45, We begin by taking the derivative of y = sin(z*) and evaluating at x = 10: ay wr) SH = conte) 4 Evaluating cos( 10,000) on a calculator (in radians) we see cos( 10,000) < 0, so we know that dy/de < 0, and therefore the function is decreasing. Next, we take the second derivative and evaluate itt x = 10: cos(e") (122°) + 4z" (inte )(Ar°) ————— wees masher | From tis we cn se that? y/d2? > 0, thos the graph concave up 4 5 + ein(on lls + sin(2at)) v MLV + ly Sein tet a cos Pat). v= 2rcos2at 47. (a) Ditferemiating gives aaa (5) “The derivative represents the rate of change ofthe depth ofthe wate in fee () The derivative, dy/dt, is zero where the tangent line 10 the curve y i horizontal. Tis occurs when dy/dt sin(t) = 0. or at ¢ = 6, 12, 18 and 24 6 am. oon. 6 pm, and midnight). When dy/dt = 0. the depth ofthe ‘ate sno longer changing. Therefore. it has either jus finshed sing ous finshed fling, and we know tha the harbors level sata maximom ora minimus. 48. (a) Differentiating, we find Rute of change of wluge _ dV" _ soe 156. Saeae = © = -1202-156sin(i20%¢) = 187208 sin(120:1) volts pe second, © ‘The rae of change of voltage with imei aro when sn(1201) of =. For example, sin(120rt) = 0 when 120s? = x, oral f = 1/120 seconds Since tee av an finite number of| Iulples ofr there ae many ies when the rat of change dV/d is zero. (o) The maximum vale of the rae of change i 18720 = 58510.6voissec 49. (a) When Et = § the springs fanhest from the clr pos A/Eeos (V/E?), so the maximum wl ocurs when ¢ = 0 = =A sin (,/Et) ste maximum seon caus when VE ‘This occurs when 120s¢ equals any multiple vn, This occurs at sme t vE 25, which isa time t= T= a a (Nera am ~ YR? since {7 > 0, an increase in the mass causes the period to increase. ‘tm 150, The tangent lines to f(2) = sin x have slope (sin 2) = eos.r. The tangent line at =O has slope f'(Q) = cos and goes through the point (0, 0). Consequently its equation is y = g(r) = x. The approximate value of sin given by ‘his equation is then g($) = § = 0.524 194 Chapter Tee SOLUTIONS Similary he tangent fine at sequently, its equation is y = H(z) 3) = Mist = 0.604. “The acta value of sin espe 8 4 and goes trough te point (2) Cone 2 + 3825. The approximate vale of sn given by this equation is then Js }. so the approximation from O is beter than that fom ‘This is because the slope of the function changes less between 3 than it does between x = $ and x =". This silustrated below. » = ote) yssing oF and cose = I ‘Thus sin x = ~ cos. sotan x = -1. The smallest z-value is = 3/4, which leads tothe smallest value sin(3z/) k 7.46 : in (22) = dyeterain i225) When: we have y @r dey d dz a SB =F (GF) = plesinn) GF = (sina + zeosa}rsinz 83. (@) Ife) in, then sine +h) — sin lima a) i cos h + sin heos.2) sim Sm tKcos = 1) + sin cose mm 7 cosh = 1 sinh sinz lim cos fim a tay Femme () S821 and () Similarly, By Aas 0. Thus, f(z) =sinz 0 +082 ve) = tion SETH) = cose sey ys jy io fim Seton = incsin h one fg SFE sin ig S6SOLUTIONS 195 54. (a) Sector OAQ is a sector of a circle with radius 2g and angle AO. Thus its atea is the left side of the inequality. Similarly the area of Sector OBR is the right side ofthe equality. The area ofthe triangle OQR is 4. tan @sinee i isa rangle wth base A tan (the segment QR) and height | (if you turn it sideways, ii easier to se this). Thus, using the given fact about areas (which i also clear from looking tthe picture), we have ae Lyrel Ae 1 = ge * (aed) SFM s «( Goya 19. yla) =a, s0g'(a) =1 a sine (ste) en ecm =sin(int) ~~ or ira) a 90) 22. B(2) 23, hl (w) = aresin w = (in2)2t92-9, 24, Note that f(z) 25, Using the chain rue gives (#) | 2 Fe)» since) (Hess) 28, Nata Thos. 9'(z) 29, Using he ut es 26. j(2) 2-5 (si sin(arctan 32) T+ 957 arcsin(sin wz) = a2. egy - Eble 208) fee O+inz? nz (ins +1n2) +22 (2) Using the chain rule gives f(z) 1 a . H'()= SSOLUTONS 137 233. Using the chain rule gives 34 snsta [(F5S4)'] = wehine 1 cost [sint(t tense) + sine(d ee Meee) | (+ cost)? Lest) [sant Sd [pee] Bsint Ta cost? 8 * aint 38, f"(2) = ~ainfaresin(s + 1))( ete) = Et Problems 236, Differentiating SG) = py tata J (2) = 2? +t = 2a? YF 42? dete Gra” Ga GHP = 2-2) “jay Since (z? +1)? > 0 forall x, we see that "(0) > 0 for L—2” > Oorz? < 1. Thatis. n(x? + 1) is concave up on the imenal -1 ne 4 = 400 and r = 10, we have 400 = 40024 at volume V = gr em? and its surface area is $ = rr® om’. Iti given that Funhermore, 0 from the chain rule, wv avjat at afar 2, the radius is increasing ata constant rate of 2 emésec. Since drat isa constant, dr fa ‘52, The volume, 1, of a cone of eighth and radius is Since the angle of the cone is =/6, sor Differentiating gives Whigs 73 To find dh fat, use the cain rule o obtain av ave dh a dha So, ah _ aV/at _ O.tmotershour _ 03 at > adh maya rh? metersaour Since r = htan(x/6) = h/ V3, we have 9S imetersour. SBSOLUTIONS — 143 53. (a) Using Pythagoras’ theorem, we see /o25-+ 9 (b) We want to calculate d2 /dt. Using the chain rule, we have dt Because he ans moving 08h, we know it te a ‘At the moment we are interesed in = = 1 km so te" 2+ VATS = ostokm de 5 M080) 45 0560-08 = 062shainn a7 yaa (We nano di, re shown nie 36 Si = stand veto o =n (2) ° do us ldr dt ~ 14 @/05p 05 at 0.8 km/min and, at the moment we are interested in, x = \/0-75. Substituting gives #1 ag G7 TeaTR as . We know that dz /at 0.4 radiansémin, Figure 36 4, Using the tiangle OSL in Figure 3.7, we label the distance 2 L 2 Onn eeeets) Figue 37 ‘We want to caleulate ce /d8. First we must find 2 asa function of 8. From the triangle, we sce $=tan@ so r= 2tand. Thus, 144 Chapter Tee SOLUTIONS . (a) Since the elevator is dewsending at 30 Tse. its height from the ground is given by h(t) = 300 ~ 30, for OS ts. (by From the triangle in he igure jg = MU = 100 _ 00 — 301 — 100 _ 200 ~ 30r ra to Tao Therelore and a 1s? © 5 (1507 + 200 = 30)? Novie tha $f i always neve. which ie reasonale singe @ creases a the elevator descends. (er Ifwe want know when 8 changes decreases the fst, we wat ond oot when d/dt has the largest magitude. in the expression for dO/de is uke smallest, or when ) = 100 feet, Le, when the elevator sa the level 200 ~ 38 of the observer. Solutions for Section 3,7 ———_ Exercises |. We ditferentate implicitly both sides ofthe equation with respect tw. dy art dye dy ee gece 2. We ilfercntiate implicitly both sides ofthe equation with respect t0 2 ae (vest) att ae) ae ty yypdt poy ae ae Ve 3. We differentiate implicitly both sides ofthe juaion with respect to. GN way! By-wedy ae ew ae = Fa 5 ‘Weean also obtain this answer by realizing that Ue orginal equation represents part of the line = 25y which has slope 1/25, 4. We differentiate implicitly both sides ofthe equation with cespeet to 2. shagt a2 STSOLUTIONS — 145 5, We differentiate implicity with respect 0 2. 6 dy rae 4 ay =0 dy & 2 ay das: — aye = 0 dy ar Fay = iy 8, We differentiate implicitly both sides ofthe equation with espect to Ine tin(y?) =3 Li lpn dy pt pg =e dy =Mr iy dz ~ Byly? ~~ 25 9.. We differentiate implicitly both sides ofthe equation with respect to 2. 1 opy aa?) ay 4 aay lt Trang Ory tg) =v + avg rey re ty: ae 1 [N+ 2'y"lly? + eye La aty yy? — ey 11, We differentiate implicitly both sides ofthe equation with espect to 2, lady, 42du 1 Pe a de 148 Chapter Tree SOLUTIONS Ee y git 22 yds ae =~ dy (2,42) _ Loeny ge (Zea) - dy (24 ay" zlny a\y )> _-ziny) = @saR) 12, We diferestiate implicitly both sides ofthe equation with respect to 2, cen) (y+) veotay) + canta tt costzy zeos(zy) * ae Ba. 2 4 dy T+ iade 32° arctan y +2° 15, Using the relation cos” y + sin? y = 1, the equation becomes: = y +2 0¢ y= —1. Hence, SH T= y42 or y= 1. Hence, =0. 16, Differentiating 2° + y* = 1 with respect ox gives Deb 2yy' =O so that tthe point (0,1) the slope is 0. 17. Differentiating sin(zy) = x with respect to 2 gives (+ xy/)eostey) = -ry'cos(rx) = 1 — yeos(zy) sothat ycostzy) Feos(ry) ‘As we move along the curve the point (1. 3). the value of dy/dz + oo, which tells us the tangent tothe curve a (1, has infinite slope: the tangent is the vertical fine 2 vf STSOLUTIONS 147 18, The slope is given by dy/d, which we find using implicit differentiation, Notice that the product rule is needed forthe second term, We uifferentiste Wo obtain: dy ‘de +g gil ty ae? +52 4 tory aye 6 dy _ ~82? tory de ~ Bete ay 4 At the point (1.2), we have dy/dz = (—3 ~ 20)/(3 +8 = 4) 23/9. 19, Differentiating with respect x gives 28/9. The slope of this curve atthe point (1.2) is Ss" 4 2ay! + 2y + 2yy' otha sean ote ve osety [At the point (1.1) the slope is ~§ 21 Fou wert fides of tenet ie “| Dieeninng imi. neh ann) i dy w+ 20h Subst ty ion yictos SH the equation for the law *iyory=$e~ 3 dy tangent Tine is y + 21, Fist we mist inte slope oie ange, at (1) Dire implicit we ae ne wy (ta t#) =? iy _ ten rie Evaluating dy/dz at (1,¢2) yields (2(1}e? — *. Using the point-slope formula forthe equation ofthe Tine, we he en pede 4 22, Fin we mut finde doe he gent #2] np ase yal lla yy _ eu-a)~# (xf +0) go a eat complexity of the above equation, we frst want to substitute 4 For x and 2 for ¥ (the coordinates of the point here we are constructing our tangent line), then solve fr 4Y Substitution iets: ply _ 2 2-9) 4 HE +2) 8G) — 10 +2) ty 22a aa 16 de

Você também pode gostar